Свойство линии регрессии минимизировать среднюю квадратическую ошибку прогноза

В предыдущих заметках предметом анализа часто становилась отдельная числовая переменная, например, доходность взаимных фондов, время загрузки Web-страницы или объем потребления безалкогольных напитков. В настоящей и следующих заметках мы рассмотрим методы предсказания значений числовой переменной в зависимости от значений одной или нескольких других числовых переменных. [1]

Материал будет проиллюстрирован сквозным примером. Прогнозирование объема продаж в магазине одежды. Сеть магазинов уцененной одежды Sunflowers на протяжении 25 лет постоянно расширялась. Однако в настоящее время у компании нет систематического подхода к выбору новых торговых точек. Место, в котором компания собирается открыть новый магазин, определяется на основе субъективных соображений. Критериями выбора являются выгодные условия аренды или представления менеджера об идеальном местоположении магазина. Представьте, что вы — руководитель отдела специальных проектов и планирования. Вам поручили разработать стратегический план открытия новых магазинов. Этот план должен содержать прогноз годового объема продаж во вновь открываемых магазинах. Вы полагаете, что торговая площадь непосредственно связана с объемом выручки, и хотите учесть этот факт в процессе принятия решения. Как разработать статистическую модель, позволяющую прогнозировать годовой объем продаж на основе размера нового магазина?

Как правило, для предсказания значений переменной используется регрессионный анализ. Его цель — разработать статистическую модель, позволяющую предсказывать значения зависимой переменной, или отклика, по значениям, по крайней мере одной, независимой, или объясняющей, переменной. В настоящей заметке мы рассмотрим простую линейную регрессию — статистический метод, позволяющий предсказывать значения зависимой переменной Y по значениям независимой переменной X. В последующих заметках будет описана модель множественной регрессии, предназначенная для предсказания значений независимой переменной Y по значениям нескольких зависимых переменных (Х1, Х2, …, Xk). [2]

Скачать заметку в формате Word или pdf, примеры в формате Excel2013

Виды регрессионных моделей

В заметке Представление числовых данных в виде таблиц и диаграмм для иллюстрации зависимости между переменными X и Y использовалась диаграмма разброса. На ней значения переменной X откладывались по горизонтальной оси, а значения переменной Y — по вертикальной. Зависимость между двумя переменными может быть разной: от самой простой до крайне сложной. Пример простейшей (линейной) зависимости показан на рис. 1.

Рис. 1. Положительная линейная зависимость

Простая линейная регрессия:

(1) Yi = β0 + β1Xi + εi

где β0 — сдвиг (длина отрезка, отсекаемого на координатной оси прямой Y), β1 — наклон прямой Y, εi— случайная ошибка переменной Y в i-м наблюдении.

В этой модели наклон β1 представляет собой количество единиц измерения переменной Y, приходящихся на одну единицу измерения переменной X. Эта величина характеризует среднюю величину изменения переменной Y (положительного или отрицательного) на заданном отрезке оси X. Сдвиг β0 представляет собой среднее значение переменной Y, когда переменная X равна 0. Последний компонент модели εi является случайной ошибкой переменной Y в i-м наблюдении. Выбор подходящей математической модели зависит от распределения значений переменных X и Y на диаграмме разброса. Различные виды зависимости переменных показаны на рис. 2.

Рис. 2. Диаграммы разброса, иллюстрирующие разные виды зависимостей

На панели А значения переменной Y почти линейно возрастают с увеличением переменной X. Этот рисунок аналогичен рис. 1, иллюстрирующему положительную зависимость между размером магазина (в квадратных футах) и годовым объемом продаж. Панель Б является примером отрицательной линейной зависимости. Если переменная X возрастает, переменная Y в целом убывает. Примером этой зависимости является связь между стоимостью конкретного товара и объемом продаж. На панели В показан набор данных, в котором переменные X и Y практически не зависят друг от друга. Каждому значению переменной X соответствуют как большие, так и малые значения переменной Y. Данные, приведенные на панели Г, демонстрируют криволинейную зависимость между переменными X и Y. Значения переменной Y возрастают при увеличении переменной X, однако скорость роста после определенных значений переменной X падает. Примером положительной криволинейной зависимости является связь между возрастом и стоимостью обслуживания автомобилей. По мере старения машины стоимость ее обслуживания сначала резко возрастает, однако после определенного уровня стабилизируется. Панель Д демонстрирует параболическую U-образную форму зависимости между переменными X и Y. По мере увеличения значений переменной X значения переменной Y сначала убывают, а затем возрастают. Примером такой зависимости является связь между количеством ошибок, совершенных за час работы, и количеством отработанных часов. Сначала работник осваивается и делает много ошибок, потом привыкает, и количество ошибок уменьшается, однако после определенного момента он начинает чувствовать усталость, и число ошибок увеличивается. На панели Е показана экспоненциальная зависимость между переменными X и Y. В этом случае переменная Y сначала очень быстро убывает при возрастании переменной X, однако скорость этого убывания постепенно падает. Например, стоимость автомобиля при перепродаже экспоненциально зависит от его возраста. Если перепродавать автомобиль в течение первого года, его цена резко падает, однако впоследствии ее падение постепенно замедляется.

Мы кратко рассмотрели основные модели, которые позволяют формализовать зависимости между двумя переменными. Несмотря на то что диаграмма разброса чрезвычайно полезна при выборе математической модели зависимости, существуют более сложные и точные статистические процедуры, позволяющие описать отношения между переменными. В дальнейшем мы будем рассматривать лишь линейную зависимость.

Вывод уравнения простой линейной регрессии

Вернемся к сценарию, изложенному в начале главы. Наша цель — предсказать объем годовых продаж для всех новых магазинов, зная их размеры. Для оценки зависимости между размером магазина (в квадратных футах) и объемом его годовых продаж создадим выборки из 14 магазинов (рис. 3).

Рис. 3. Площади и годовые объемы продаж 14 магазинов сети Sunflowers: (а) исходные данные; (б) диаграмма разброса

Анализ рис. 3 показывает, что между площадью магазина X и годовым объемом продаж Y существует положительная зависимость. Если площадь магазина увеличивается, объем продаж возрастает почти линейно. Таким образом, наиболее подходящей для исследования является линейная модель. Остается лишь определить, какая из линейных моделей точнее остальных описывает зависимость между анализируемыми переменными.

Метод наименьших квадратов

Данные, представленные на рис. 1а, получены для случайной выборки магазинов. Если верны некоторые предположения (об этом чуть позже), в качестве оценки параметров генеральной совокупности (β0 и β1) можно использовать сдвиг b0 и наклон b1 прямой Y. Таким образом, уравнение простой линейной регрессии принимает следующий вид:

где — предсказанное значение переменной Y для i-гo наблюдения, Xi — значение переменной X в i-м наблюдении.

Для того чтобы предсказать значение переменной Y, в уравнении (2) необходимо определить два коэффициента регрессии — сдвиг b0 и наклон b1 прямой Y. Вычислив эти параметры, проведем прямую на диаграмме разброса. Затем исследователь может визуально оценить, насколько близка регрессионная прямая к точкам наблюдения. Простая линейная регрессия позволяет найти прямую линию, максимально приближенную к точкам наблюдения. Критерии соответствия можно задать разными способами. Возможно, проще всего минимизировать разности между фактическими значениями Yi, и предсказанными значениями . Однако, поскольку эти разности могут быть как положительными, так и отрицательными, следует минимизировать сумму их квадратов.

Поскольку = b0 + b1Xi, сумма квадратов принимает следующий вид:

Параметры b0 и b1 неизвестны. Таким образом, сумма квадратов разностей является функцией, зависящей от сдвига b0 и наклона b1 выборки Y. Для того чтобы найти значения параметров b0 и b1, минимизирующих сумму квадратов разностей, применяется метод наименьших квадратов. При любых других значениях сдвига b0 и наклона b1 сумма квадратов разностей между фактическими значениями переменной Y и ее наблюдаемыми значениями лишь увеличится.

До того, как Excel взял на себя всю рутинную работу, вычисления по методу наименьших квадратов были очень трудоемкими. Excel позволяет решать подобные задачи двумя способами. Во-первых, можно воспользоваться Пакетом анализа (строка Регрессия). Результаты представлены на рис. 4. Во-вторых, можно, выделив точки на графике (как на рис. 3б), кликнуть правой кнопкой мыши и выбрать Добавить линию тренда. Далее можно выбрать вид линии тренда (в нашем случае – Линейная), отформатировать линию, показать на графике уравнение и величину достоверности аппроксимации (R2) (рис. 5).

Рис. 4. Результаты решения задачи о зависимости между площадями и годовыми объемами продаж в магазинах сети Sunflower (получены с помощью Пакета анализа Excel)

Рис. 5. Диаграмма разброса и линия регрессии (тренда) в задаче о выборе магазина

Как следует из рис. 4 и 5, b0 = 0,9645, а b1 = 1,6699. Таким образом, уравнение линейной регрессии для этих данных имеет следующий вид: = 0,9645 + 1,6699Xi. Вычисленный наклон b1 = +1,6699. Это означает, что при возрастании переменной X на единицу среднее значение переменной Y возрастает на 1,6699 единиц. Иначе говоря, увеличение площади магазина на один квадратный фут приводит к увеличению годового объема продаж на 1,67 тыс. долл. Таким образом, наклон представляет собой долю годового объема продаж, зависящую от размера магазина. Вычисленный сдвиг b0 = +0,9645 (млн. долл.). Эта величина представляет собой среднее значение переменной Y при X = 0. Поскольку площадь магазина не может равняться нулю, сдвиг можно считать долей годового дохода, зависящей от других факторов. Следует отметить, однако, что сдвиг переменной Y выходит за пределы диапазона переменной X. Следовательно, к интерпретации параметра b0 необходимо относиться внимательно.

Пример 1. Один экономист решил предсказать изменение индекса 500 наиболее активно покупаемых акций на Нью-Йоркской фондовой бирже, публикуемого агентством Standard and Poor, на основе показателей экономики США за 50 лет. В результате он получил следующее уравнение линейной регрессии: Ŷi = –5,0 + 7Хi. Какой смысл имеют параметры сдвига b0 и наклона b1.

Решение. Сдвиг регрессии b0 равен –5,0. Это значит, что если рост экономики США равен нулю, индекс акций за год снизится на 5%. Наклон b1 равен 7. Следовательно, при увеличении темпов роста экономики на 1% индекс акций возрастает на 7%.

Пример 2. Вернемся к сценарию, изложенному в начале заметки. Применим модель линейной регрессии для прогноза объема годовых продаж во всех новых магазинах в зависимости от их размеров. Предположим, что площадь магазина равна 4000 квадратных футов. Какой среднегодовой объем продаж можно прогнозировать?

Решение. Подставим значение X = 4 (тыс. кв. футов) в уравнение линейной регрессии: = 0,9645 + 1,6699Xi = 0,9645 + 1,6699*4 = 7,644 млн. долл. Итак, прогнозируемый среднегодовой объем продаж в магазине, площадь которого равна 4000 кв. футов, составляет 7 644 000 долл.

Прогнозирование в регрессионном анализе: интерполяция и экстраполяция

Применяя регрессионную модель для прогнозирования, необходимо учитывать лишь допустимые значения независимой переменной. В этот диапазон входят все значения переменной X, начиная с минимальной и заканчивая максимальной. Таким образом, предсказывая значение переменной Y при конкретном значении переменной X, исследователь выполняет интерполяцию между значениями переменной X в диапазоне возможных значений. Однако экстраполяция значений за пределы этого интервала не всегда релевантна. Например, пытаясь предсказать среднегодовой объем продаж в магазине, зная его площадь (рис. 3а), мы можем вычислять значение переменной Y лишь для значений X от 1,1 до 5,8 тыс. кв. футов. Следовательно, прогнозировать среднегодовой объем продаж можно лишь для магазинов, площадь которых не выходит за пределы указанного диапазона. Любая попытка экстраполяции означает, что мы предполагаем, будто линейная регрессия сохраняет свой характер за пределами допустимого диапазона.

Оценки изменчивости

Вычисление сумм квадратов. Для того чтобы предсказать значение зависимой переменной по значениям независимой переменной в рамках избранной статистической модели, необходимо оценить изменчивость. Существует несколько способов оценки изменчивости. Первый способ использует общую сумму квадратов (total sum of squares — SST), позволяющую оценить колебания значений Yi вокруг среднего значения . В регрессионном анализе полная вариация, представляющая собой полную сумму квадратов, разделяется на объяснимую вариацию, или сумму квадратов регрессии (regression sum of squares — SSR), и необъяснимую вариацию, или сумму квадратов ошибок (error sum of squares — SSE). Объяснимая вариация характеризует взаимосвязь между переменными X и Y, а необъяснимая зависит от других факторов (рис. 6).

Рис. 6. Оценки изменчивости в модели регрессии

Сумма квадратов регрессии (SSR) представляет собой сумму квадратов разностей между Ŷi (предсказанным значением переменной Y) и (средним значением переменной Y). Сумма квадратов ошибок (SSE) является частью вариации переменной Y, которую невозможно описать с помощью регрессионной модели. Эта величина зависит от разностей между наблюдаемыми и предсказанными значениями.

Полная сумма квадратов (SST) равна сумме квадратов регрессии плюс сумма квадратов ошибок:

(3)   SST = SSR + SSE

Полная сумма квадратов (SST) равна сумме квадратов разностей между наблюдаемыми значениями переменной Y и ее средним значением:

Сумма квадратов регрессии (SSR) равна сумме квадратов разностей между предсказанными значениями переменной Y и ее средним значением:

Сумма квадратов ошибок (SSE) равна сумме квадратов разностей между наблюдаемыми и предсказанными значениями переменной Y:

Суммы квадратов, вычисленные с помощью программы Пакета анализа Excel при решении задачи о сети магазинов Sunflowers, представлены на рис. 4.

Полная сумма квадратов разностей равна SST = 116,9543. Эта величина состоит из суммы квадратов регрессии (SSR) равной 105,7476, и суммы квадратов ошибок (SSE), равной 11,2067.

Коэффициент смешанной корреляции. Величины SSR, SSE и SST не имеют очевидной интерпретации. Однако отношение суммы квадратов регрессии (SSR) к полной сумме квадратов (SST) представляет собой оценку полезности регрессионного уравнения. Это отношение называется коэффициентом смешанной корреляции r2:

Коэффициент смешанной корреляции оценивает долю вариации переменной Y, которая объясняется независимой переменной X в регрессионной модели. В задаче о сети магазинов Sunflowers SSR = 105,7476 и SST = 116,9543. Следовательно, r2 = 105,7476 / 116,9543 = 0,904. Таким образом, 90,4% вариации годового объема продаж объясняется изменчивостью площади магазинов, измеренной в квадратных футах. Данная величина r2 свидетельствует о сильной положительной линейной взаимосвязи между двумя переменными, поскольку применение регрессионной модели снижает изменчивость прогнозируемых годовых объемов продаж на 90,4%. Только 9,6% изменчивости годовых объемов продаж в выборке магазинов объясняются другими факторами, не учтенными в регрессионной модели.

Коэффициент смешанной корреляции в задаче о сети магазинов Sunflowers представлен в таблице Регрессионная статистика на рис. 4.

Среднеквадратичная ошибка оценки. Хотя метод наименьших квадратов позволяет вычислить линию, минимизирующую отклонение от наблюдаемых значений, наличие суммы квадратов ошибок (SSE) свидетельствует о том, что линейная регрессия не дает абсолютной точности прогноза, если, конечно, точки наблюдения не лежат на регрессионной прямой. Однако ожидать этого так же неестественно, как предполагать, что все выборочные значения точно равны их среднему арифметическому. Следовательно, необходима статистика, которая позволила бы оценить отклонение предсказанных значений переменной Y от ее реальных значений, аналогично тому, как стандартное отклонение, введенное ранее, позволяет оценить колебание данных вокруг их средней величины. Стандартное отклонение наблюдаемых значений переменной Y от ее регрессионной прямой называется среднеквадратичной ошибкой оценки. Отклонение реальных данных от регрессионной прямой в задаче о сети магазинов Sunflowers показано на рис. 5.

Среднеквадратичная ошибка оценки

где Yi — фактическое значение переменной Y при заданном значении Xi, Ŷi — предсказанное значение переменной Y при заданном значении Xi, SSE — сумма квадратов ошибок.

Поскольку SSE = 11,2067, по формуле (8) получаем:

Таким образом, среднеквадратичная ошибка оценки равна 0,9664 млн. долл. (т.е. 966 400 долл.). Этот параметр также рассчитывается Пакетом анализа (см. рис. 4). Среднеквадратичная ошибка оценки характеризует отклонение реальных данных от линии регрессии. Она измеряется в тех же единицах, что и переменная Y. По смыслу среднеквадратичная ошибка очень похожа на стандартное отклонение. В то время как стандартное отклонение характеризует разброс данных вокруг их среднего значения, среднеквадратичная ошибка позволяет оценить колебание точек наблюдения вокруг регрессионной прямой. Cреднеквадратичная ошибка оценки позволяет обнаружить статистически значимую зависимость, существующую между двумя переменными, и предсказать значения переменной Y.

Предположения

Обсуждая методы проверки гипотез и дисперсионного анализа, мы не раз подчеркивали важность условий, которые должны обеспечивать корректность сделанных выводов. Поскольку и регрессионный, и дисперсионный анализ используют линейную модель, условия их применения приблизительно одинаковы:

  • Ошибка должна иметь нормальное распределение.
  • Вариация данных вокруг линии регрессии должна быть постоянной.
  • Ошибки должны быть независимыми.

Первое предположение, о нормальном распределении ошибок, требует, чтобы при каждом значении переменной X ошибки линейной регрессии имели нормальное распределение (рис. 7). Как и t— и F-критерий дисперсионного анализа, регрессионный анализ довольно устойчив к нарушениям этого условия. Если распределение ошибок относительно линии регрессии при каждом значении X не слишком сильно отличается от нормального, выводы относительно линии регрессии и коэффициентов регрессии изменяются незначительно.

Рис. 7. Предположение о нормальном распределении ошибок

Второе условие заключается в том, что вариация данных вокруг линии регрессии должна быть постоянной при любом значении переменной X. Это означает, что величина ошибки как при малых, так и при больших значениях переменной X должна изменяться в одном и том же интервале (см. рис. 7). Это свойство очень важно для метода наименьших квадратов, с помощью которого определяются коэффициенты регрессии. Если это условие нарушается, следует применять либо преобразование данных, либо метод наименьших квадратов с весами.

Третье предположение, о независимости ошибок, заключается в том, что ошибки регрессии не должны зависеть от значения переменной X. Это условие особенно важно, если данные собираются на протяжении определенного отрезка времени. В этих ситуациях ошибки, присущие конкретному отрезку времени, часто коррелируют с ошибками, характерными для предыдущего периода.

Анализ остатков

Чуть выше при решении задачи о сети магазинов Sunflowers мы использовали модель линейной регрессии. Рассмотрим теперь анализ ошибок — графический метод, позволяющий оценить точность регрессионной модели. Кроме того, с его помощью можно обнаружить потенциальные нарушения условий применения регрессионного анализа.

Оценка пригодности эмпирической модели. Остаток, или оценка ошибки еi, представляет собой разность между наблюдаемым (Yi) и предсказанным (Ŷi) значениями зависимой переменной при заданном значении Xi.

(9) ei = Yi – Ŷi

Для оценки пригодности эмпирической модели регрессии остатки откладываются по вертикальной оси, а значения Xi — по горизонтальной. Если эмпирическая модель пригодна, график не должен иметь ярко выраженной закономерности. Если же модель регрессии не пригодна, на рисунке проявится зависимость между значениями Xi и остатками еi.

Рассмотрим примеры (рис. 8). Панель А иллюстрирует возрастание переменной Y при увеличении переменной X. Однако зависимость между этими переменными носит нелинейный характер, поскольку скорость возрастания переменной Y падает при увеличении переменной X. Таким образом, для аппроксимации зависимости между этими переменными лучше подойдет квадратичная модель. Особенно ярко квадратичная зависимость между величинами Xi и ei проявляется на панели Б. Графическое изображение остатков позволяет отфильтровать или удалить линейную зависимость между переменными X и Y и выявить недостаточную точность модели простой линейной регрессии. Таким образом, в данной ситуации вместо простой линейной модели должна применяться квадратичная модель, обладающая более высокой точностью.

Рис. 8. Исследование эмпирической модели простой линейной регрессии

Вернемся к задаче о сети магазинов Sunflowers и посмотрим, хорошо ли подходит простая линейная регрессия для ее решения. Соответствующие данные и расчеты приведены на рис. 9а (формулы можно посмотреть в Excel-файле). Построим диаграмму разброса, откладывая по вертикальной оси остатки ei, а по горизонтальной — независимую переменную Xi (рис. 9б). Несмотря на большой разброс остатков, между ei и Хi нет ярко выраженной зависимости. Остатки одинаково часто принимают как положительные, так и отрицательные значения. Это позволяет сделать вывод, что модель линейной регрессии пригодна для решения задачи о сети магазинов Sunflowers.

Рис. 9. Остатки ei, вычисленные при решении задачи о сети магазинов Sunflowers

Значения остатков (таблица на рис. 9а) и график остатков (аналог рис. 9б) можно получить непосредственно в процедуре Регрессия Пакета анализа. Просто поставьте соответствующие галки (рис. 10).

Рис. 10. Остатки ei и график остатков полученные с помощью Пакета анализа

Проверка условий. График остатков позволяет оценить вариации ошибок. На рис. 10 нет особых различий между ошибками, соответствующими разным значениям Xi. Следовательно, вариации ошибок при разных значениях Хi приблизительно одинаковы. Рассмотрим гипотетическую ситуацию, в которой это условие не выполняется (рис. 11). На этом рисунке изображен эффект веера: при возрастании значений Хi ошибки увеличиваются. Таким образом, изменчивость значений Yi при разных значениях Хi является непостоянной.

Рис. 11. Пример нарушения условия независимости вариаций ошибок от Xi

Нормальность. Чтобы проверить предположение о нормальном распределении ошибок, построим график нормального распределения на основе точечного графика, на вертикальной оси которого отложены значения остатков, а на горизонтальной оси — соответствующие квантили стандартизованного нормального распределения (подробнее см. Проверка гипотезы о нормальном распределении). Для построения такого графика значения остатков должны быть упорядочены по возрастанию (рис. 12). График нормального распределения может быть построен одним кликом с помощью Пакета анализа Excel – просто поставьте соответствующую галочку в окне Регрессия (см. рис. 10, самый низ окна Регрессия – опция График нормальной вероятности).

Рис. 12. График нормального распределения для остатков

Без визуализации данных (с помощью гистограммы, диаграммы «ствол и листья», блочной диаграммы или графика как на рис. 12) проверить предположение о нормальном распределении ошибок очень трудно. Данные, изображенные на рис. 12, не слишком сильно отличаются от нормального распределения. Устойчивость регрессионного анализа и небольшой объем выборки позволяют утверждать, что условие о нормальном распределении ошибок нарушается незначительно.

Независимость. Предположение о независимости ошибок также проверяется с помощью графика остатков. Данные, собранные на протяжении некоторого периода времени, иногда демонстрируют эффект автокорреляции между последовательными наблюдениями. В таких ситуациях остатки зависят от значений предыдущих остатков. Подобная связь между остатками нарушает предположение о независимости ошибок. Эффект автокорреляции хорошо выявляется на графике. Кроме того, его можно измерить с помощью процедуры Дурбина-Уотсона (см. ниже). Если данные о размерах магазинов и объемах продаж собирались в течение одного и того же периода времени, гипотезу об их независимости проверять не имеет смысла.

Измерение автокорреляции: статистика Дурбина–Уотсона

Одним из основных предположений о регрессионной модели является гипотеза о независимости ее ошибок. Если данные собираются в течение определенного отрезка времени, это условие часто нарушается, поскольку остаток в определенный момент времени может оказаться приблизительно равным предыдущим остаткам. Такое поведение остатков называется автокорреляцией. Если набор данных обладает свойством автокорреляции, корректность регрессионной модели становится весьма сомнительной.

Распознавание автокорреляции с помощью графика остатков. Для выявления автокорреляции необходимо упорядочить остатки по времени и построить их график. Если данные обладают положительной автокорреляцией, на графике возникнут кластеры остатков, имеющие одинаковый знак. В случае отрицательной автокорреляции остатки будут скачкообразно принимать то положительные, то отрицательные значения. Этот вид автокорреляции очень редко встречается в регрессионном анализе, поэтому мы рассмотрим лишь положительную автокорреляцию. Проиллюстрируем ее следующим примером. Предположим, что менеджер магазина, доставляющего товары на дом, пытается предсказать объем продаж по количеству клиентов, совершивших покупки в течение 15 недель (рис. 13).

Рис. 13. Количество клиентов и объемы продаж за 15 недель

Поскольку данные собирались на протяжении 15 последовательных недель в одном и том же магазине, необходимо определить, наблюдается ли эффект автокорреляции. Построим регрессию с использованием Пакета анализа; включим вывод Остатков, но не будем включать График остатков (рис. 14).

Рис. 14. Параметры линейной регрессии, полученные с использованием Пакета анализа

Анализ рис. 14 показывает, что r2 = 0,657. Это значит, что 65,7% вариации объемов продаж объясняется изменчивостью количества клиентов. Кроме того, сдвиг b0 переменной Y равен –16,032, а наклон b1 = 0,0308. Однако, прежде чем применять эту модель, необходимо выполнить анализ остатков. Поскольку данные собирались на протяжении 15 последовательных недель, их следует отобразить на графике в том же порядке (рис. 15).

Рис. 15. Зависимость остатков от времени

Анализ рис. 15 показывает, что остатки циклически колеблются вверх и вниз. Эта цикличность является явным признаком автокорреляции. Следовательно, гипотезу о независимости остатков следует отклонить.

Статистика Дурбина-Уотсона. Автокорреляцию можно выявить и измерить с помощью статистики Дурбина-Уотсона. Эта статистика оценивает корреляцию между соседними остатками:

где еi — остаток, соответствующий i-му периоду времени.

Чтобы лучше понять статистику Дурбина-Уотсона, рассмотрим ее составные части. Числитель представляет собой сумму квадратов разностей между соседними остатками, начиная со второго и заканчивая n-м наблюдением. Знаменатель является суммой квадратов остатков. Вот, что по этому поводу написано в Википедии:

где ρ1 – коэффициент автокорреляции; если ρ1 = 0 (нет автокорреляции), D ≈ 2; если ρ1 ≈ 1 (положительная автокорреляции), D ≈ 0; если ρ1 = -1 (отрицательная автокорреляции), D ≈ 4.

На практике применение критерия Дурбина-Уотсона основано на сравнении величины D с критическими теоретическими значениями dL и dU для заданного числа наблюдений n, числа независимых переменных модели k (для простой линейной регрессии k = 1) и уровня значимости α. Если D < dL, гипотеза о независимости случайных отклонений отвергается (следовательно, присутствует положительная автокорреляция); если D > dU, гипотеза не отвергается (то есть автокорреляция отсутствует); если dL < D < dU, нет достаточных оснований для принятия решения. Когда расчётное значение D превышает 2, то с dL и dU сравнивается не сам коэффициент D, а выражение (4 – D).

Для вычисления статистики Дурбина-Уотсона в Excel обратимся к нижней таблице на рис. 14 Вывод остатка. Числитель в выражении (10) вычисляется с помощью функции =СУММКВРАЗН(массив1;массив2), а знаменатель =СУММКВ(массив) (рис. 16).

Рис. 16. Формулы расчета статистики Дурбина-Уотсона

В нашем примере D = 0,883. Основной вопрос заключается в следующем — какое значение статистики Дурбина-Уотсона следует считать достаточно малым, чтобы сделать вывод о существовании положительной автокорреляции? Необходимо соотнести значение D с критическими значениями (dL и dU), зависящими от числа наблюдений n и уровня значимости α (рис. 17).

Рис. 17. Критические значения статистики Дурбина-Уотсона (фрагмент таблицы)

Таким образом, в задаче об объеме продаж в магазине, доставляющем товары на дом, существуют одна независимая переменная (k = 1), 15 наблюдений (n = 15) и уровень значимости α = 0,05. Следовательно, dL= 1,08 и dU = 1,36. Поскольку D = 0,883 < dL= 1,08, между остатками существует положительная автокорреляция, метод наименьших квадратов применять нельзя.

Проверка гипотез о наклоне и коэффициенте корреляции

Выше регрессия применялась исключительно для прогнозирования. Для определения коэффициентов регрессии и предсказания значения переменной Y при заданной величине переменной X использовался метод наименьших квадратов. Кроме того, мы рассмотрели среднеквадратичную ошибку оценки и коэффициент смешанной корреляции. Если анализ остатков подтверждает, что условия применимости метода наименьших квадратов не нарушаются, и модель простой линейной регрессии является адекватной, на основе выборочных данных можно утверждать, что между переменными в генеральной совокупности существует линейная зависимость.

Применение t-критерия для наклона. Проверяя, равен ли наклон генеральной совокупности β1 нулю, можно определить, существует ли статистически значимая зависимость между переменными X и Y. Если эта гипотеза отклоняется, можно утверждать, что между переменными X и Y существует линейная зависимость. Нулевая и альтернативная гипотезы формулируются следующим образом: Н0: β1 = 0 (нет линейной зависимости), Н1: β1 ≠ 0 (есть линейная зависимость). По определению t-статистика равна разности между выборочным наклоном и гипотетическим значением наклона генеральной совокупности, деленной на среднеквадратичную ошибку оценки наклона:

(11) t = (b1β1) / Sb1

где b1 – наклон прямой регрессии по выборочным данным, β1 – гипотетический наклон прямой генеральной совокупности, , а тестовая статистика t имеет t-распределение с n – 2 степенями свободы.

Проверим, существует ли статистически значимая зависимость между размером магазина и годовым объемом продаж при α = 0,05. t-критерий выводится наряду с другими параметрами при использовании Пакета анализа (опция Регрессия). Полностью результаты работы Пакета анализа приведены на рис. 4, фрагмент, относящийся к t-статистике – на рис. 18.

Рис. 18. Результаты применения t-критерия, полученные с помощью Пакета анализа Excel

Поскольку число магазинов n = 14 (см. рис.3), критическое значение t-статистики при уровне значимости α = 0,05 можно найти по формуле: tL =СТЬЮДЕНТ.ОБР(0,025;12) = –2,1788, где 0,025 – половина уровня значимости, а 12 = n – 2; tU =СТЬЮДЕНТ.ОБР(0,975;12) = +2,1788.

Поскольку t-статистика = 10,64 > tU = 2,1788 (рис. 19), нулевая гипотеза Н0 отклоняется. С другой стороны, р-значение для Х = 10,6411, вычисляемое по формуле =1-СТЬЮДЕНТ.РАСП(D3;12;ИСТИНА), приближенно равно нулю, поэтому гипотеза Н0 снова отклоняется. Тот факт, что р-значение почти равно нулю, означает, что если бы между размерами магазинов и годовым объемом продаж не существовало реальной линейной зависимости, обнаружить ее с помощью линейной регрессии было бы практически невозможно. Следовательно, между средним годовым объемом продаж в магазинах и их размером существует статистически значимая линейная зависимость.

Рис. 19. Проверка гипотезы о наклоне генеральной совокупности при уровне значимости, равном 0,05, и 12 степенях свободы

Применение F-критерия для наклона. Альтернативным подходом к проверке гипотез о наклоне простой линейной регрессии является использование F-критерия. Напомним, что F-критерий применяется для проверки отношения между двумя дисперсиями (подробнее см. Однофакторный дисперсионный анализ). При проверке гипотезы о наклоне мерой случайных ошибок является дисперсия ошибки (сумма квадратов ошибок, деленная на количество степеней свободы), поэтому F-критерий использует отношение дисперсии, объясняемой регрессией (т.е. величины SSR, деленной на количество независимых переменных k), к дисперсии ошибок (MSE = SYX2).

По определению F-статистика равна среднему квадрату отклонений, обусловленных регрессией (MSR), деленному на дисперсию ошибки (MSE): F = MSR/MSE, где MSR = SSR / k, MSE = SSE/(n– k – 1), k – количество независимых переменных в регрессионной модели. Тестовая статистика F имеет F-распределение с k и n – k – 1 степенями свободы.

При заданном уровне значимости α решающее правило формулируется так: если F > FU, нулевая гипотеза отклоняется; в противном случае она не отклоняется. Результаты, оформленные в виде сводной таблицы дисперсионного анализа, приведены на рис. 20.

Рис. 20. Таблица дисперсионного анализа для проверки гипотезы о статистической значимости коэффициента регрессии

Аналогично t-критерию F-критерий выводится в таблицу при использовании Пакета анализа (опция Регрессия). Полностью результаты работы Пакета анализа приведены на рис. 4, фрагмент, относящийся к F-статистике – на рис. 21.

Рис. 21. Результаты применения F-критерия, полученные с помощью Пакета анализа Excel

F-статистика равна 113,23, а р-значение близко к нулю (ячейка Значимость F). Если уровень значимости α равен 0,05, определить критическое значение F-распределения с одной и 12 степенями свободы можно по формуле FU =F.ОБР(1-0,05;1;12) = 4,7472 (рис. 22). Поскольку F = 113,23 > FU = 4,7472, причем р-значение близко к 0 < 0,05, нулевая гипотеза Н0 отклоняется, т.е. размер магазина тесно связан с его годовым объемом продаж.

Рис. 22. Проверка гипотезы о наклоне генеральной совокупности при уровне значимости, равном 0,05, с одной и 12 степенями свободы

Доверительный интервал, содержащий наклон β1. Для проверки гипотезы о существовании линейной зависимости между переменными можно построить доверительный интервал, содержащий наклон β1 и убедиться, что гипотетическое значение β1 = 0 принадлежит этому интервалу. Центром доверительного интервала, содержащего наклон β1, является выборочный наклон b1, а его границами — величины b1 ± tn–2Sb1

Как показано на рис. 18, b1 = +1,670, n = 14, Sb1 = 0,157. t12 =СТЬЮДЕНТ.ОБР(0,975;12) = 2,1788. Следовательно, b1 ± tn–2Sb1 = +1,670 ± 2,1788 * 0,157 = +1,670 ± 0,342, или + 1,328 ≤ β1 ≤ +2,012. Таким образом, наклон генеральной совокупности с вероятностью 0,95 лежит в интервале от +1,328 до +2,012 (т.е. от 1 328 000 до 2 012 000 долл.). Поскольку эти величины больше нуля, между годовым объемом продаж и площадью магазина существует статистически значимая линейная зависимость. Если бы доверительный интервал содержал нуль, между переменными не было бы зависимости. Кроме того, доверительный интервал означает, что каждое увеличение площади магазина на 1 000 кв. футов приводит к увеличению среднего объема продаж на величину от 1 328 000 до 2 012 000 долларов.

Использование t-критерия для коэффициента корреляции. Ранее был введен коэффициент корреляции r, представляющий собой меру зависимости между двумя числовыми переменными. С его помощью можно установить, существует ли между двумя переменными статистически значимая связь. Обозначим коэффициент корреляции между генеральными совокупностями обеих переменных символом ρ. Нулевая и альтернативная гипотезы формулируются следующим образом: Н0: ρ = 0 (нет корреляции), Н1: ρ ≠ 0 (есть корреляция). Проверка существования корреляции:

где r = +, если b1 > 0, r = –, если b1 < 0. Тестовая статистика t имеет t-распределение с n – 2 степенями свободы.

В задаче о сети магазинов Sunflowers r2 = 0,904, а b1— +1,670 (см. рис. 4). Поскольку b1 > 0, коэффициент корреляции между объемом годовых продаж и размером магазина равен r = +√0,904 = +0,951. Проверим нулевую гипотезу, утверждающую, что между этими переменными нет корреляции, используя t-статистику:

При уровне значимости α = 0,05 нулевую гипотезу следует отклонить, поскольку t = 10,64 > 2,1788. Таким образом, можно утверждать, что между объемом годовых продаж и размером магазина существует статистически значимая связь.

При обсуждении выводов, касающихся наклона генеральной совокупности, доверительные интервалы и критерии для проверки гипотез являются взаимозаменяемыми инструментами. Однако вычисление доверительного интервала, содержащего коэффициент корреляции, оказывается более сложным делом, поскольку вид выборочного распределения статистики r зависит от истинного коэффициента корреляции.

Оценка математического ожидания и предсказание индивидуальных значений

В этом разделе рассматриваются методы оценки математического ожидания отклика Y и предсказания индивидуальных значений Y при заданных значениях переменной X.

Построение доверительного интервала. В примере 2 (см. выше раздел Метод наименьших квадратов) регрессионное уравнение позволило предсказать значение переменной Y при заданном значении переменной X. В задаче о выборе места для торговой точки средний годовой объем продаж в магазине площадью 4000 кв. футов был равен 7,644 млн. долл. Однако эта оценка математического ожидания генеральной совокупности является точечной. Ранее для оценки математического ожидания генеральной совокупности была предложена концепция доверительного интервала. Аналогично можно ввести понятие доверительного интервала для математического ожидания отклика при заданном значении переменной X:

где , = b0 + b1Xi – предсказанное значение переменное Y при X = Xi, SYX – среднеквадратичная ошибка, n – объем выборки, Xi — заданное значение переменной X, µY|X=Xi – математическое ожидание переменной Y при Х = Хi, SSX =

Анализ формулы (13) показывает, что ширина доверительного интервала зависит от нескольких факторов. При заданном уровне значимости возрастание амплитуды колебаний вокруг линии регрессии, измеренное с помощью среднеквадратичной ошибки, приводит к увеличению ширины интервала. С другой стороны, как и следовало ожидать, увеличение объема выборки сопровождается сужением интервала. Кроме того, ширина интервала изменяется в зависимости от значений Xi. Если значение переменной Y предсказывается для величин X, близких к среднему значению , доверительный интервал оказывается уже, чем при прогнозировании отклика для значений, далеких от среднего.

Допустим, что, выбирая место для магазина, мы хотим построить 95%-ный доверительный интервал для среднего годового объема продаж во всех магазинах, площадь которых равна 4000 кв. футов:

Следовательно, средний годовой объем продаж во всех магазинах, площадь которых равна 4 000 кв. футов, с 95% -ной вероятностью лежит в интервале от 6,971 до 8,317 млн. долл.

Вычисление доверительного интервала для предсказанного значения. Кроме доверительного интервала для математического ожидания отклика при заданном значении переменной X, часто необходимо знать доверительный интервал для предсказанного значения. Несмотря на то что формула для вычисления такого доверительного интервала очень похожа на формулу (13), этот интервал содержит предсказанное значение, а не оценку параметра. Интервал для предсказанного отклика YX=Xi при конкретном значении переменной Xi определяется по формуле:

Предположим, что, выбирая место для торговой точки, мы хотим построить 95%-ный доверительный интервал для предсказанного годового объема продаж в магазине, площадь которого равна 4000 кв. футов:

Следовательно, предсказанный годовой объем продаж в магазине, площадь которого равна 4000 кв. футов, с 95%-ной вероятностью лежит в интервале от 5,433 до 9,854 млн. долл. Как видим, доверительный интервал для предсказанного значения отклика намного шире, чем доверительный интервал для его математического ожидания. Это объясняется тем, что изменчивость при прогнозировании индивидуальных значений намного больше, чем при оценке математического ожидания.

Подводные камни и этические проблемы, связанные с применением регрессии

Трудности, связанные с регрессионным анализом:

  • Игнорирование условий применимости метода наименьших квадратов.
  • Ошибочная оценка условий применимости метода наименьших квадратов.
  • Неправильный выбор альтернативных методов при нарушении условий применимости метода наименьших квадратов.
  • Применение регрессионного анализа без глубоких знаний о предмете исследования.
  • Экстраполяция регрессии за пределы диапазона изменения объясняющей переменной.
  • Путаница между статистической и причинно-следственной зависимостями.

Широкое распространение электронных таблиц и программного обеспечения для статистических расчетов ликвидировало вычислительные проблемы, препятствовавшие применению регрессионного анализа. Однако это привело к тому, что регрессионный анализ стали применять пользователи, не обладающие достаточной квалификацией и знаниями. Откуда пользователям знать об альтернативных методах, если многие из них вообще не имеют ни малейшего понятия об условиях применимости метода наименьших квадратов и не умеют проверять их выполнение?

Исследователь не должен увлекаться перемалыванием чисел — вычислением сдвига, наклона и коэффициента смешанной корреляции. Ему нужны более глубокие знания. Проиллюстрируем это классическим примером, взятым из учебников. Анскомб показал, что все четыре набора данных, приведенных на рис. 23, имеют одни и те же параметры регрессии (рис. 24).

Рис. 23. Четыре набора искусственных данных

Рис. 24. Регрессионный анализ четырех искусственных наборов данных; выполнен с помощью Пакета анализа (кликните на рисунке, чтобы увеличить изображение)

Итак, с точки зрения регрессионного анализа все эти наборы данных совершенно идентичны. Если бы анализ был на этом закончен, мы потеряли бы много полезной информации. Об этом свидетельствуют диаграммы разброса (рис. 25) и графики остатков (рис. 26), построенные для этих наборов данных.

Рис. 25. Диаграммы разброса для четырех наборов данных

Диаграммы разброса и графики остатков свидетельствуют о том, что эти данные отличаются друг от друга. Единственный набор, распределенный вдоль прямой линии, — набор А. График остатков, вычисленных по набору А, не имеет никакой закономерности. Этого нельзя сказать о наборах Б, В и Г. График разброса, построенный по набору Б, демонстрирует ярко выраженную квадратичную модель. Этот вывод подтверждается графиком остатков, имеющим параболическую форму. Диаграмма разброса и график остатков показывают, что набор данных В содержит выброс. В этой ситуации необходимо исключить выброс из набора данных и повторить анализ. Метод, позволяющий обнаруживать и исключать выбросы из наблюдений, называется анализом влияния. После исключения выброса результат повторной оценки модели может оказаться совершенно иным. Диаграмма разброса, построенная по данным из набора Г, иллюстрирует необычную ситуацию, в которой эмпирическая модель значительно зависит от отдельного отклика (Х8 = 19, Y8 = 12,5). Такие регрессионные модели необходимо вычислять особенно тщательно. Итак, графики разброса и остатков являются крайне необходимым инструментом регрессионного анализа и должны быть его неотъемлемой частью. Без них регрессионный анализ не заслуживает доверия.

Рис. 26. Графики остатков для четырех наборов данных

Как избежать подводных камней при регрессионном анализе:

  • Анализ возможной взаимосвязи между переменными X и Y всегда начинайте с построения диаграммы разброса.
  • Прежде чем интерпретировать результаты регрессионного анализа, проверяйте условия его применимости.
  • Постройте график зависимости остатков от независимой переменной. Это позволит определить, насколько эмпирическая модель соответствует результатам наблюдения, и обнаружить нарушение постоянства дисперсии.
  • Для проверки предположения о нормальном распределении ошибок используйте гистограммы, диаграммы «ствол и листья», блочные диаграммы и графики нормального распределения.
  • Если условия применимости метода наименьших квадратов не выполняются, используйте альтернативные методы (например, модели квадратичной или множественной регрессии).
  • Если условия применимости метода наименьших квадратов выполняются, необходимо проверить гипотезу о статистической значимости коэффициентов регрессии и построить доверительные интервалы, содержащие математическое ожидание и предсказанное значение отклика.
  • Избегайте предсказывать значения зависимой переменной за пределами диапазона изменения независимой переменной.
  • Имейте в виду, что статистические зависимости не всегда являются причинно-следственными. Помните, что корреляция между переменными не означает наличия причинно-следственной зависимости между ними.

Резюме. Как показано на структурной схеме (рис. 27), в заметке описаны модель простой линейной регрессии, условия ее применимости и способы проверки этих условий. Рассмотрен t-критерий для проверки статистической значимости наклона регрессии. Для предсказания значений зависимой переменной использована регрессионная модель. Рассмотрен пример, связанный с выбором места для торговой точки, в котором исследуется зависимость годового объема продаж от площади магазина. Полученная информация позволяет точнее выбрать место для магазина и предсказать его годовой объем продаж. В следующих заметках будет продолжено обсуждение регрессионного анализа, а также рассмотрены модели множественной регрессии.

Рис. 27. Структурная схема заметки

Предыдущая заметка Критерий согласия «хи-квадрат»

Следующая заметка Введение в множественную регрессию

К оглавлению Статистика для менеджеров с использованием Microsoft Excel


[1] Используются материалы книги Левин и др. Статистика для менеджеров. – М.: Вильямс, 2004. – с. 792–872

[2] Если зависимая переменная является категорийной, необходимо применять логистическую регрессию.

Your question seems to imply that least squares regression is the only method to fit a linear model. As mentioned in other answers, there are other perfectly legitimate methods that can be used to fit a linear predictor. A common thread of these methods is that they are tractable, i.e.: there are concrete steps that can be taken to find the actual solution (or rather, an approximation to the solution within some acceptable tolerance).

Tractability is not an intrinsic property of the method. What is tractable at any given time depends on the state of technological developments. If some day quantum computing becomes part of standard technological development, then the list of tractable methods will be greatly expanded.

In times of Gauss and Euler, the list of tractable methods was far more limited than our current list, and least squares was a technological advance with lasting consequences.

A second important quality of whatever method one chooses is effectiveness. Gauss use of least squares helped him make important accurate predictions in the context of astronomical observations. I speculate that faced with Gauss success, researchers wanted to know how he did it, rather than why he did what he did.

A third feature of fitting methods that tilts the scale in favour of least squares is interpretability. We seek models to abstract patterns from observations, so that we can understand differences and make predictions. The theoretical framework of least squares provides guidance in model building. Lately I had the chance to apply minimization of the sum of the absolute values of the error (with a quadratic penalty on the size of the parameters), known as LASSO. The model was selected by crossvalidation. None of the niceties regarding significance of the coefficients, which are at the core of least squares, are immediately available. For some time, much of observational sciences consisted of finding statistically significance in model parameters, because the producing a model was the goal of data analysis. Of late, there has been an increasing interest in using models to prediction expected outcomes, which have resulted in the addition of various model fitting methods to the analyst tool-box.

Since I digressed, I will summarize my answer:
* least squares regression is not the only method in use to fit linear models
* the methods in use are those that are tractable (can be implemented), effective (solve the problem at hand), yield interpretable results (when the need arises)

As for the popularity of least squares:
* from the mid 1700’s to time of wide-spread availability of computing machines, least squares regression was the state of the art in linear model fitting (disregard the objections of the Bayesians, they had conjugate pairs, but not until the late 20th century they could handle more general parameter priors)
* least-squares regression, when it assumptions are met, provides a framework that can be use for guidance in model building

Now I’ll digress again by addressing objections:
Objector: …but least squares minimizes a function that is differentiable..
Answer1: So? Convex minimization is well developed, numerical methods are available
Answer2: it is 2016, enough with eighteenth century technology

Objector: …but p-values, where art thou?
Answer: if you need p-values to publish, then use least-squares. You can also use other methods of model fitting, and estimate the distribution of parameter estimates through, for example, bootstrapping. If what you need are predictions, then you need not worry about p-values. Use statistical methods to ensure your models are stable, and the results reproducible. The importance of p-values in the scientific literature has been overplayed, either by dishonesty or ignorance. The loss of significance or strength of relations in successive repetitions of many experiments is a well documented fact, cause by p-value significance driven models.

Objector: … but all the hordes trained in least squares…?
Answer: (speechless)

Objector: …but should we dispose of least-squares in our model building pursues?
Answer: No. There is nothing intrinsically wrong with least-squares. It applies when the hypothesis underlying the method hold (namely, Gaussian distribution of the remainders, iid observations), and in any case, least squares gives the BLUE, which is all you often need.

Hope that helps. thanks for the question.

Так как значения
известны без ошибок, а значениянезависимы и равноточны, то оценка
дисперсии вычисляется по формуле:

,
где
,
(23)

–фактические
значения результативного признака,
полученного по данным наблюдений,
– значения результативного признака,
рассчитанного по уравнению регрессии
и полученного подстановкой значений
факторного признака в уравнение
регрессии:.
В нашем примере.

Средняя квадратическая
ошибка уравнения регрессии:
.

Для нахождения
оценки дисперсии
величины
составим таблицу:

1

6,97

3

362,51

359,176771

11,1104159

33,3312477

2

7,40

6

394,91

398,6374926

13,89420071

83,36520426

3

7,83

2

459,71

440,5026538

368,92215

737,8442999

4

8,26

14

484,01

484,7722546

0,581031999

8,134447986

5

8,69

14

529,14

531,446295

5,318996396

74,46594955

6

9,12

24

579,185

580,524775

1,794996917

43,079926

7

9,55

14

633,28

632,0076946

1,618761158

22,66265621

8

9,98

11

693,87

685,8950538

63,59976769

699,5974446

9

10,41

10

736,73

742,1868526

29,77723975

297,7723975

10

10,84

2

799,91

800,883091

0,946905997

1,893811994

.

Средняя квадратическая
ошибка уравнения регрессии

.

Сравним полученную
величину со средним квадратическим
отклонением результативного признака
,
получим,
т.е.,
следовательно, использование уравнения
регрессии является целесообразным.

2.10. Интервальные оценки параметров квадратичной линии регрессии генеральной совокупности

Доверительные
интервалы для коэффициентов
при заданной доверительной вероятностиимеют вид:,
гдеопределяется из таблицы для закона
распределения Стьюдента по выходным
величинами числу степеней свободы.

В данном случае
,,
отсюда.

Оценки
коэффициентов

определяются формулами

,

где
,– определитель системы (22),– алгебраическое дополнение элементав определителе.

;

;

59,78703801;

;
;

;

.

;

;

;

;

;

;

.

;

;

;

;

;

;

.

2.11. Нахождение коэффициента детерминации

Коэффициент
детерминации, интегрально характеризующий
точностные свойства уравнения регрессии,
определяем по формуле (21).

,
,,

.

Сравним
с.,
следовательно, полученная регрессионная
модель работоспособна.

2.12. Проверка адекватности регрессионной модели

Проверка адекватности
модели возможна только при
,
где– число опытов (),– число оцениваемых коэффициентов
регрессии математической модели ().
В нашем случае,
следовательно, можно проводить проверку
адекватности.

Найдем дисперсию
адекватности
,

где

;
.

Получим
.

Найдем
,где
;

.

Найдем
,
где– уровень значимости,– число степеней свободы дисперсии
адекватности,– число степеней свободы дисперсии
воспроизводимости.

Сравним
и,.

Построенная модель
считается адекватной и может быть
использована для описания объекта.

Список литературы:

  1. Гмурман
    В.Е. Теория вероятностей и математическая
    статистика: учеб. пособие для вузов.
    М.: Высш. образование, 2008. – 479 с.

  2. Гмурман
    В.Е. Руководство к решению задач по
    теории вероятностей и математической
    статистике: учеб. пособие для вузов.
    М.: Высш. образование, 2009. – 404
    с.

  3. Виленкин
    Н.Я., Потапов В.Г. Задачник-практикум по
    теории вероятностей с элементами
    комбинаторики и математической
    статистики. М.: Просвещение, 1979. – 112 с.

  4. Кремер
    Н.Ш. Теория вероятностей и математическая
    статистика. М.: ЮНИТИ-ДАНА, 2006. – 573 с.

АКТУАЛЬНОСТЬ ТЕМЫ

Общие положения

Про регрессионный анализ вообще, и его применение в DataScience написано очень много. Есть множество учебников, монографий, справочников и статей по прикладной статистике, огромное количество информации в интернете, примеров расчетов. Можно найти множество кейсов, реализованных с использованием средств Python. Казалось бы — что тут еще можно добавить?

Однако, как всегда, есть нюансы:

1. Регрессионный анализ — это прежде всего процесс, набор действий исследователя по определенному алгоритму: «подготовка исходных данных — построение модели — анализ модели — прогнозирование с помощью модели». Это ключевая особенность. Не представляет особой сложности сформировать DataFrame исходных данных и построить модель, запустить процедуру из библиотеки statsmodels. Однако подготовка исходных данных и последующий анализ модели требуют гораздо больших затрат человеко-часов специалиста и строк программного кода, чем, собственно, построение модели. На этих этапах часто приходится возвращаться назад, корректировать модель или исходные данные. Этому, к сожалению, во многих источниках, не удаляется достойного внимания, а иногда — и совсем не уделяется внимания, что приводит к превратному представлению о регрессионном анализе.

2. Далеко не во всех источниках уделяется должное внимание интерпретации промежуточных и финальных результатов. Специалист должен уметь интерпретировать каждую цифру, полученную в ходе работы над моделью.

3. Далеко не все процедуры на этапах подготовки исходных данных или анализа модели в источниках разобраны подробно. Например, про проверку значимости коэффициента детерминации найти информацию не представляет труда, а вот про проверку адекватности модели, построение доверительных интервалов регрессии или про специфические процедуры (например, тест Уайта на гетероскедастичность) информации гораздо меньше.

4. Своеобразная сложность может возникнуть с проверкой статистических гипотез: для отечественной литературы по прикладной статистике больше характерно проверять гипотезы путем сравнения расчетного значения критерия с табличным, а в иностранных источниках чаще определяется расчетный уровень значимости и сравнивается с заданным (чаще всего 0.05 = 1-0.95). В разных источниках информации реализованы разные подходы. Инструменты python (прежде всего библиотеки scipy и statsmodels) также в основном оперируют с расчетным уровнем значимости.

5. Ну и, наконец, нельзя не отметить, что техническая документация библиотеки statsmodels составлена, на мой взгляд, далеко не идеально: информация излагается путано, изобилует повторами и пропусками, описание классов, функций и свойств выполнено фрагментарно и количество примеров расчетов — явно недостаточно.

Поэтому я решил написать ряд обзоров по регрессионному анализу средствами Python, в которых акцент будет сделан на практических примерах, алгоритме действий исследователя, интерпретации всех полученных результатов, конкретных методических рекомендациях. Буду стараться по возможности избегать теории (хотя совсем без нее получится) — все-таки предполагается, что специалист DataScience должен знать теорию вероятностей и математическую статистику, хотя бы в рамках курса высшей математики для технического или экономического вуза.

В данном статье остановимся на самои простом, классическом, стереотипном случае — простой линейной регрессии (simple linear regression), или как ее еще принято называть — парной линейной регрессионной модели (ПЛРМ) — в ситуации, когда исследователя не подстерегают никакие подводные камни и каверзы — исходные данные подчиняются нормальному закону, в выборке отсутствуют аномальные значения, отсутствует ложная корреляция. Более сложные случаи рассмотрим в дальнейшем.

Для построение регрессионной модели будем пользоваться библиотекой statsmodels.

В данной статье мы рассмотрим по возможности полный набор статистических процедур. Некоторые из них (например, дескриптивная статистика или дисперсионный анализ регрессионной модели) могут показаться избыточными. Все так, но эти процедуры улучшают наше представление о процессе и об исходных данных, поэтому в разбор я их включил, а каждый исследователь сам вправе для себя определить, потребуются ему эти процедуры или нет.

Краткий обзор источников

Источников информации по корреляционному и регрессионному анализу огромное количество, в них можно просто утонуть. Поэтому позволю себе просто порекомендовать ряд источников, на мой взгляд, наиболее полезных:

  1. Кобзарь А.И. Прикладная математическая статистика. Для инженеров и научных работников. — М.: ФИЗМАТЛИТ, 2006. — 816 с.

  2. Львовский Е.Н. Статистические методы построения эмпирических формул. — М.: Высшая школа, 1988. — 239 с.

  3. Фёрстер Э., Рёнц Б. Методы корреляционного и регрессионного анализа / пер с нем. — М.: Финансы и статистика, 1983. — 302 с.

  4. Афифи А., Эйзен С. Статистический анализ. Подход с использованием ЭВМ / пер с англ. — М.: Мир, 1982. — 488 с.

  5. Дрейпер Н., Смит Г. Прикладной регрессионный анализ. Книга 1 / пер.с англ. — М.: Финансы и статистика, 1986. — 366 с.

  6. Айвазян С.А. и др. Прикладная статистика: Исследование зависимостей. — М.: Финансы и статистика, 1985. — 487 с.

  7. Прикладная статистика. Основы эконометрики: В 2 т. 2-е изд., испр. — Т.2: Айвазян С.А. Основы эконометрики. — М.: ЮНИТИ-ДАНА, 2001. — 432 с.

  8. Магнус Я.Р. и др. Эконометрика. Начальный курс — М.: Дело, 2004. — 576 с.

  9. Носко В.П. Эконометрика. Книга 1. — М.: Издательский дом «Дело» РАНХиГС, 2011. — 672 с.

  10. Брюс П. Практическая статистика для специалистов Data Science / пер. с англ. — СПб.: БХВ-Петербург, 2018. — 304 с.

  11. Уатт Дж. и др. Машинное обучение: основы, алгоритмы и практика применения / пер. с англ. — СПб.: БХВ-Петербург, 2022. — 640 с.

Прежде всего следует упомянуть справочник Кобзаря А.И. [1] — это безусловно выдающийся труд. Ничего подобного даже близко не издавалось. Всем рекомендую иметь под рукой.

Есть очень хорошее практическое пособие [2] — для начинающих и практиков.>

Добротная работа немецких авторов [3]. Все разобрано подробно, обстоятельно, с примерами — очень хорошая книга. Примеры приведены из области экономики.

Еще одна добротная работа — [4], с примерами медико-биологического характера.

Работа [5] считается одним из наиболее полных изложений прикладного регрессионного анализа.

Более сложные работы — [6] (классика жанра), [7], [8], [9] — выдержаны на достаточно высоком математическом уровне, примеры из экономической области.

Свежие работы [10] (с примерами на языке R) и [11] (с примерами на python).

Cтатьи

Статей про регрессионный анализ в DataScience очень много, обращаю внимание на некоторые весьма полезные из них.

Серия статей «Python, корреляция и регрессия», охватывающая весь процесс регрессионного анализа:

  • первичная обработка данных, визуализация и корреляционный анализ;

  • регрессия;

  • теория матриц в регрессионном анализе, проверка  адекватности, мультиколлинеарность;

  • прогнозирование с помощью регрессионных моделей.

Очень хороший обзор «Интерпретация summary из statsmodels для линейной регрессии». В этой статье даны очень полезные ссылки:

  • Statistical Models

  • Interpreting Linear Regression Through statsmodels .summary()

Статья «Регрессионные модели в Python».

Основные предпосылки (гипотезы) регрессионного анализа

Очень кратко — об этом написано тысячи страниц в учебниках — но все же вспомним некоторые основы теории.

Проверка исходных предпосылок является очень важным моментом при статистическом анализе регрессионной модели. Если мы рассматриваем классическую линейную регрессионную модель вида:

то основными предпосылками при использовании обычного метода наименьших квадратов (МНК) для оценки ее параметров являются:

  1. Среднее значение (математическое ожидание) случайной составляющей равно нулю:

  1. Дисперсия случайной составляющей является постоянной:

В случае нарушения данного условия мы сталкиваемся с явлением гетероскедастичности.

  1. Значения случайной составляющей статистически независимы (некоррелированы) между собой:

В случае нарушения данного условия мы сталкиваемся с явлением автокорреляции.

  1. Условие существования обратной матрицы

что эквивалентно одному из двух следующих условий:

то есть число наблюдений должно превышать число параметров.

  1. Значения случайной составляющей некоррелированы со значениями независимых переменных:

  1. Случайная составляющая имеет нормальный закон распределения (с математическим ожиданием равным нулю — следует из условия 1):

Более подробно — см.: [3, с.90], [4, с.147], [5, с.122], [6, с.208], [7, с.49], [8, с.68], [9, с.88].

Кроме гетероскедастичности и автокорреляции возможно возникновение и других статистических аномалий — мультиколлинеарности, ложной корреляции и т.д.

Доказано, что оценки параметров, полученные с помощью МНК, обладают наилучшими свойствами (несмещенность, состоятельность, эффективность) при соблюдении ряда условий:

  • выполнение приведенных выше исходных предпосылок регрессионного анализа;

  • число наблюдений на одну независимую переменную должно быть не менее 5-6;

  • должны отсутствовать аномальные значения (выбросы).

Кроме обычного МНК существуют и другие его разновидности (взвешенный МНК, обобщенный МНК), которые применяются при наличии статистических аномалий. Кроме МНК применяются и другие методы оценки параметров моделей. В этом обзоре мы эти вопросы рассматривать не будем.

Алгоритм проведения регрессионного анализа

Алгоритм действий исследователя при построении регрессионной модели (полевые работы мы, по понятным причинам, не рассматриваем — считаем, что исходные данные уже получены):

  1. Подготовительный этап — постановка целей и задач исследования.

  2. Первичная обработка исходных данных — об этом много написано в учебниках и пособиях по DataScience, сюда могут относится:

  • выявление нерелевантных признаков (признаков, которые не несут полезной информации), нетипичных данных (выбросов), неинформативных признаков (имеющих большое количество одинаковых значений) и работа с ними (удаление/преобразование);

  • выделение категориальных признаков;

  • работа с пропущенными значениями;

  • преобразование признаков-дат в формат datetime и т.д.

  1. Визуализация исходных данных — предварительный графический анализ.

  2. Дескриптивная (описательная) статистика — расчет выборочных характеристик и предварительные выводы о свойствах исходных данных.

  3. Исследование закона распределения исходных данных и, при необходимости, преобразование исходных данных к нормальному закону распределения.

  4. Выявление статистически аномальных значений (выбросов), принятие решения об их исключении.

    Этапы 4, 5 и 6 могут быть при необходимости объединены.

  5. Корреляционный анализ — исследование корреляционных связей между исходными данными; это разведка перед проведением регрессионного анализа.

  6. Построение регрессионной модели:

  • выбор моделей;

  • выбор методов;

  • оценка параметров модели.

  1. Статистический анализ регрессионной модели:  

  • оценка ошибок аппроксимации (error metrics);

  • анализ остатков (проверка нормальности распределения остатков и гипотезы о равенстве нулю среднего значения остатков);

  • проверка адекватности модели;

  • проверка значимости коэффициента детерминации;

  • проверка значимости коэффициентов регрессии;

  • проверка мультиколлинеарности (для множественных регрессионных моделей; вообще мультиколлинеарные переменные выявляются еще на стадии корреляционного анализа);

  • проверка автокорреляции;

  • проверка гетероскедастичности.

   Этапы 8 и 9 могут быть при необходимости повторяться несколько раз.

  1. Сравнительный анализ нескольких регрессионных моделей, выбор наилучшей (при необходимости).

  2. Прогнозирование с помощью регрессионной модели и оценка качества прогноза.

  3. Выводы и рекомендации.

Само собой, этот алгоритм не есть истина в последней инстанции — в зависимости от особенностей исходных данных и вида модели могут возникать дополнительные задачи.

Применение пользовательских функций

Далее в обзоре мной будут использованы несколько пользовательских функций для решения разнообразных задач. Все эти функции созданы для облегчения работы и уменьшения размера программного кода. Данные функции загружается из пользовательского модуля my_module__stat.py, который доступен в моем репозитории на GitHub. Лично мне так удобнее работать, хотя каждый исследователь сам формирует себе инструменты по душе — особенно в части визуализации. Желающие могут пользоваться этими функциями, либо создать свои.

Итак, вот перечень данных функций:

  • graph_scatterplot_sns — функция позволяет построить точечную диаграмму средствами seaborn и сохранить график в виде png-файла;

  • graph_hist_boxplot_probplot_XY_sns  — функция позволяет визуализировать исходные данные для простой линейной регрессии путем одновременного построения гистограммы, коробчатой диаграммы и вероятностного графика (для переменных X и Y) средствами seaborn и сохранить график в виде png-файла; имеется возможность выбирать, какие графики строить (h — hist, b — boxplot, p — probplot);

  • descriptive_characteristics — функция возвращает в виде DataFrame набор статистических характеристики выборки, их ошибок и доверительных интервалов;

  • detecting_outliers_mad_test — функция выполняет проверку наличия аномальных значений (выбросов) по критерию наибольшего абсолютного отклонения (более подробно — см.[1, с.547]);

  • norm_distr_check — проверка нормальности распределения исходных данных с использованием набора из нескольких статистических тестов;

  • corr_coef_check — функция выполняет расчет коэффициента линейной корреляции Пирсона, проверку его значимости и расчет доверительных интервалов; об этой функции я писал в своей статье.

  • graph_regression_plot_sns —  — функция позволяет построить график регрессионной модели.

Ряд пользовательских функций мы создаем в процессе данного обзора (они тоже включены в пользовательский модуль my_module__stat.py):

  • regression_error_metrics — расчет ошибок аппроксимации регрессионной модели;

  • ANOVA_table_regression_model — вывод таблицы дисперсионного анализа регрессионной модели;

  • regression_model_adequacy_check — проверка адекватности регрессионной модели по критерию Фишера;

  • determination_coef_check — проверка значимости коэффициента детерминации по критерию Фишера;

  • regression_coef_check — проверка значимости коэффициентов регрессии по критеирю Стьюдента;

  • Goldfeld_Quandt_test, Breush_Pagan_test, White_test — проверка гетероскедастичности с использование тестов Голдфелда-Квандта, Бриша-Пэгана и Уайта соответственно;

  • regression_pair_predict — функция для прогнозирования с помощью парной регрессионной модели: рассчитывает прогнозируемое значение переменной Y по заданной модели, а также доверительные интервалы среднего и индивидуального значения для полученного прогнозируемого значения Y;

  • graph_regression_pair_predict_plot_sns — прогнозирование: построение графика регрессионной модели (с доверительными интервалами) и вывод расчетной таблицы с данными для заданной области значений X.

ПОСТАНОВКА ЗАДАЧИ

В качестве примера рассмотрим практическую задачу из области экспертизы промышленной безопасности — калибровку ультразвукового прибора для определения прочности бетона.

Итак, суть задачи: при обследовании несущих конструкций зданий и сооружений эксперт определяет прочность бетона с использованием ультразвукового прибора «ПУЛЬСАР-2.1», для которого необходимо предварительно построить градуировочную зависимость. Заключается это в следующем — производятся замеры с фиксацией следующих показателей:

  • X — показания ультразвукового прибора «ПУЛЬСАР-2.1» (м/с)

  • Y — результаты замера прочности бетона (методом отрыва со скалыванием) склерометром ИПС-МГ4.03.

Предполагается, что между показателями X и Y имеется линейная регрессионная зависимость, которая позволит прогнозировать прочность бетона на основании измерений, проведенных прибором «ПУЛЬСАР-2.1».

Были выполнены замеры фактической прочности бетона конструкций для бетонов одного вида с одним типом крупного заполнителя, с единой технологией производства. Для построения были выбраны 14 участков (не менее 12), включая участки, в которых значение косвенного показателя максимальное, минимальное и имеет промежуточные значения.

Настройка заголовков отчета:

# Общий заголовок проекта
Task_Project = 'Калибровка ультразвукового прибора "ПУЛЬСАР-2.1" \nдля определения прочности бетона'

# Заголовок, фиксирующий момент времени
AsOfTheDate = ""

  # Заголовок раздела проекта
Task_Theme = ""

# Общий заголовок проекта для графиков
Title_String = f"{Task_Project}\n{AsOfTheDate}"

# Наименования переменных
Variable_Name_X = "Скорость УЗК (м/с)"
Variable_Name_Y = "Прочность бетона (МПа)"

# Константы
INCH = 25.4    # мм/дюйм
  DecPlace = 5    # number of decimal places - число знаков после запятой

# Доверительная вероятность и уровень значимости:
p_level = 0.95
a_level = 1 - p_level   

Подключение модулей и библиотек:

# Стандартные модули и библиотеки

import os    # загрузка модуля для работы с операционной системой
import sys
import platform
print('{:<35}{:^0}'.format("Текущая версия Python: ", platform.python_version()), '\n')

import math
from math import *    # подключаем все содержимое модуля math, используем без псевдонимов

import numpy as np
#print ("Текущая версия модуля numpy: ", np.__version__)
print('{:<35}{:^0}'.format("Текущая версия модуля numpy: ", np.__version__))
from numpy import nan

import scipy as sci
print('{:<35}{:^0}'.format("Текущая версия модуля scipy: ", sci.__version__))
import scipy.stats as sps

import pandas as pd
print('{:<35}{:^0}'.format("Текущая версия модуля pandas: ", pd.__version__))

import matplotlib as mpl
print('{:<35}{:^0}'.format("Текущая версия модуля matplotlib: ", mpl.__version__))
import matplotlib.pyplot as plt

import seaborn as sns
print('{:<35}{:^0}'.format("Текущая версия модуля seaborn: ", sns.__version__))

import statsmodels.api as sm
import statsmodels.formula.api as smf
import statsmodels.graphics.api as smg
import statsmodels.stats.api as sms
from statsmodels.compat import lzip
print('{:<35}{:^0}'.format("Текущая версия модуля statsmodels: ", sm.__version__))

import statistics as stat    # module 'statistics' has no attribute '__version__'

import sympy as sym
print('{:<35}{:^0}'.format("Текущая версия модуля sympy: ", sym.__version__))

# Настройки numpy
np.set_printoptions(precision = 4, floatmode='fixed')

# Настройки Pandas
pd.set_option('display.max_colwidth', None)    # текст в ячейке отражался полностью вне зависимости от длины
pd.set_option('display.float_format', lambda x: '%.4f' % x)

# Настройки seaborn
sns.set_style("darkgrid")
sns.set_context(context='paper', font_scale=1, rc=None)    # 'paper', 'notebook', 'talk', 'poster', None

# Настройки Mathplotlib
f_size = 8    # пользовательская переменная для задания базового размера шрифта
plt.rcParams['figure.titlesize'] = f_size + 12    # шрифт заголовка
plt.rcParams['axes.titlesize'] = f_size + 10      # шрифт заголовка
plt.rcParams['axes.labelsize'] = f_size + 6       # шрифт подписей осей
plt.rcParams['xtick.labelsize'] = f_size + 4      # шрифт подписей меток
plt.rcParams['ytick.labelsize'] = f_size + 4
plt.rcParams['legend.fontsize'] = f_size + 6      # шрифт легенды

# Пользовательские модули и библиотеки

Text1 = os.getcwd()    # вывод пути к текущему каталогу
#print(f"Текущий каталог: {Text1}")

sys.path.insert(1, "D:\REPOSITORY\MyModulePython")

from my_module__stat import *

ФОРМИРОВАНИЕ ИСХОДНЫХ ДАННЫХ

Показания ультразвукового прибора «ПУЛЬСАР-2.1» (м/с):

X = np.array([
    4416, 4211, 4113, 4110, 4122,
    4427, 4535, 4311, 4511, 4475,
    3980, 4490, 4007, 4426
    ])

Результаты замера прочности бетона (методом отрыва со скалыванием) прибором ИПС-МГ4.03:

Y = np.array([
    34.2, 35.1, 31.5, 30.8, 30.0,
    34.0, 35.4, 35.8, 38.0, 37.7,
    30.0, 37.8, 31.0, 35.2
    ])

Запишем данные в DataFrame:

calibrarion_df = pd.DataFrame({
    'X': X,
    'Y': Y})
display(calibrarion_df)
calibrarion_df.info()

Сохраняем данные в csv-файл:

calibrarion_df.to_csv(
    path_or_buf='data/calibrarion_df.csv',
    mode='w+',
    sep=';')

Cоздаем копию исходной таблицы для работы:

dataset_df = calibrarion_df.copy()

ВИЗУАЛИЗАЦИЯ ДАННЫХ

Границы значений переменных (при построении графиков):

(Xmin_graph, Xmax_graph) = (3800, 4800)
(Ymin_graph, Ymax_graph) = (25, 45)
# Пользовательская функция
graph_scatterplot_sns(
    X, Y,
    Xmin=Xmin_graph, Xmax=Xmax_graph,
    Ymin=Ymin_graph, Ymax=Ymax_graph,
    color='orange',
    title_figure=Task_Project,
    x_label=Variable_Name_X,
    y_label=Variable_Name_Y,
    s=100,
    file_name='graph/scatterplot_XY_sns.png')

Существует универсальный набор графиков — гистограмма, коробчатая диаграмма, вероятностный график — которые позволяют исследователю сделать предварительные выводы о свойствах исходных данных.

Так как объем выборки невелик (n=14), строить гистограммы распределения переменных X и Y не имеет смысла, поэтому ограничимся построением коробчатых диаграмм и вероятностных графиков:

# Пользовательская функция
graph_hist_boxplot_probplot_XY_sns(
    data_X=X, data_Y=Y,
    data_X_min=Xmin_graph, data_X_max=Xmax_graph,
    data_Y_min=Ymin_graph, data_Y_max=Ymax_graph,  
    graph_inclusion='bp',    # выбираем для построения виды графиков: b - boxplot, p - probplot)
    data_X_label=Variable_Name_X,
    data_Y_label=Variable_Name_Y,
    title_figure=Task_Project,
    file_name='graph/hist_boxplot_probplot_XY_sns.png')    

Для сравнения характера распределений переменных X и Y возможно также построить совмещенную коробчатую диаграмму по стандартизованным данным:

# стандартизуем исходные данные
standardize_df = lambda X: ((X - np.mean(X))/np.std(X))

dataset_df_standardize = dataset_df.copy()
dataset_df_standardize = dataset_df_standardize.apply(standardize_df)
display(dataset_df_standardize)

# построим график
fig, axes = plt.subplots(figsize=(210/INCH, 297/INCH/2))
axes.set_title("Распределение стандартизованных переменных X и Y", fontsize = 16)
sns.boxplot(
    data=dataset_df_standardize,    
    orient='h',
    width=0.5,
    ax=axes)
plt.show()

Графический анализ позволяет сделать следующие выводы:

  1. Отсутствие выбросов на коробчатых диаграммах свидетельствует об однородности распределения переменных.

  2. Смещение медианы вправо на коробчатых диаграммах свидетельствует о левосторонней асимметрии распределения.

ДЕСКРИПТИВНАЯ (ОПИСАТЕЛЬНАЯ СТАТИСТИКА)

Собственно говоря, данный этап требуется проводить далеко не всегда, однако с помощью статистических характеристик выборки мы тоже можем сделать полезные выводы.

Описательная статистика исходных данных средствами библиотеки Pandas — самый простой вариант:

dataset_df.describe()

Описательная статистика исходных данных средствами библиотеки statsmodels — более развернутый вариант, с большим количеством показателей:

from statsmodels.stats.descriptivestats import Description
result = Description(
    dataset_df,
    stats=["nobs", "missing", "mean", "std_err", "ci", "ci", "std", "iqr", "mad", "coef_var", "range", "max", "min", "skew", "kurtosis", "mode",
           "median", "percentiles", "distinct", "top", "freq"],
    alpha=a_level,
    use_t=True)
display(result.summary())

Описательная статистика исходных данных с помощью пользовательской функции descriptive_characteristics:

# Пользовательская функция
descriptive_characteristics(X)

Выводы:

  1. Сравнение показателей среднего арифметического (mean) и медианы (median) свидетельствует о левосторонней асимметрии (т.к.mean < median).

  2. Значение коэффициента вариации CV = 0.0445 и доверительный интервал для него 0.0336 ≤ CV ≤ 0.0657 свидетельствует об однородности исходных данных (т.к. CV ≤ 0.33).

  3. Значение показателя асимметрии skew (As) = -0.3101 свидетельствует об умеренной левосторонней асимметрии распределении (т.к. |As| ≤ 0.5, As < 0).

  4. Значение показателя эксцесса kurtosis (Es) = -1.4551 свидетельствует о плосковершинном распределении (platykurtic distribution) (т.к. Es < 0).

# Пользовательская функция
descriptive_characteristics(Y)

Выводы:

  1. Сравнение показателей среднего арифметического (mean) и медианы (median) свидетельствует о левосторонней асимметрии (т.к.mean < median).

  2. Значение коэффициента вариации CV = 0.0822 и доверительный интервал для него 0.06202 ≤ CV ≤ 0.1217 свидетельствует об однородности исходных данных (т.к. CV ≤ 0.33).

  3. Значение показателя асимметрии skew (As) = -0.1109 свидетельствует о приблизительно симметричном распределении (т.к. |As| ≤ 0.25).

  4. Значение показателя эксцесса kurtosis (Es) = -1.3526 свидетельствует о плосковершинном распределении (platykurtic distribution) (т.к. Es < 0).

ПРОВЕРКА НОРМАЛЬНОСТИ РАСПРЕДЕЛЕНИЯ

Для проверки нормальности распределения использована пользовательская функция norm_distr_check, которая объединяет в себе набор стандартных статистических тестов проверки нормальности. Все тесты относятся к стандартному инструментарию Pyton (библиотека scipy, модуль stats), за исключением теста Эппса-Палли (Epps-Pulley test); о том, как реализовать этот тест средствами Pyton я писал в своей статье https://habr.com/ru/post/685582/.

Примечание: для использования функции norm_distr_check в каталог с ipynb-файлом необходимо поместить папку table c файлом Tep_table.csv, который содержит табличные значения статистики критерия Эппса-Палли.

# пользовательская функция
norm_distr_check(X)

# Пользовательская функция
norm_distr_check (Y)

Вывод: большинство статистических тестов позволяют принять гипотезу о нормальности распределения переменных X и Y.

ПРОВЕРКА АНОМАЛЬНЫХ ЗНАЧЕНИЙ (ВЫБРОСОВ)

Статистическую проверку аномальных значений (выбросов) не стоит путать с проверкой выбросов, которая проводится на этапе первичной обработки результатов наблюдений. Последняя проводится с целью отсеять явные ошибочные данные (например, в результате неправильно поставленной запятой величина показателя может увеличиться/уменьшиться на порядок); здесь же мы говорим о статистической проверке данных, которые уже прошли этап первичной обработки.

Имеется довольно много критериев для проверки аномальных значений (подробнее см.[1]); вообще данная процедура довольно неоднозначная:

  • критерии зависят от вида распределения;

  • мало данных о сравнительной мощности этих критериев;

  • даже в случае принятии гипотезы о нормальном распределении в выборке могут быть обнаружены аномальные значения и пр.

Кроме существует дилемма: если какие-то значения в выборке признаны выбросами — стоит или не стоит исследователю исключать их? Ведь каждое значение несет в себе информацию, причем иногда весьма ценную, а сильно отклоняющиеся от основного массива данные (которые не являются выбросами в смысле первичной обработки, но являются статистическим значимыми аномальными значениями) могут кардинально изменить статистический вывод.

В общем, о задаче выявления аномальных значений (выбросов) можно написать отдельно, а пока, в данном разборе, ограничимся проверкой аномальных значений по критерию наибольшего максимального отклонения (см.[1, с.547]) с помощью пользовательской функции detecting_outliers_mad_test. Данные функция возвращает DataFrame, которые включает список аномальных значений со следующими признаками:

  • value — проверяемое значение из выборки;

  • mad_calc и mad_table — расчетное и табличное значение статистики критерия;

  • outlier_conclusion — вывод (выброс или нет).

Обращаю внимание, что критерий наибольшего максимального отклонения можно использовать только для нормально распределенных данных.

# пользовательская функция
print("Проверка наличия выбросов переменной X:\n")
result = detecting_outliers_mad_test(X)
mask = (result['outlier_conclusion'] == 'outlier')
display(result[mask])

# пользовательская функция
print("Проверка наличия выбросов переменной Y:\n")
result = detecting_outliers_mad_test(Y)
mask = (result['outlier_conclusion'] == 'outlier')
display(result[mask])

Вывод: в случае обеих переменных X и Y список пуст, следовательно, аномальных значений (выбросов) не выявлено.

КОРРЕЛЯЦИОННЫЙ АНАЛИЗ

Корреляционный анализ — это разведка перед построением регрессионной модели.

Выполним расчет коэффициента линейной корреляции Пирсона, проверку его значимости и построение доверительных интервалов с помощью пользовательской функции corr_coef_check (про эту функцию более подробно написано в моей статье https://habr.com/ru/post/683442/):

# пользовательская функция
display(corr_coef_check(X, Y, scale='Evans'))

Выводы:

  1. Значение коэффициента корреляции coef_value = 0.8900 свидетельствует о весьма сильной корреляционной связи (по шкале Эванса).

  2. Коэффициент корреляции значим по критерию Стьюдента: t_calc ≥ t_table, a_calc ≤ a_level.

  3. Доверительный интервал для коэффициента корреляции: 0.6621 ≤ coef_value ≤ 0.9625.

РЕГРЕССИОННЫЙ АНАЛИЗ

Предварительная визуализация

python позволяет выполнить предварительную визуализацию, например, с помощью функции jointplot библиотеки seaborn:

fig = plt.figure(figsize=(297/INCH, 210/INCH))
axes = sns.jointplot(
    x=X, y=Y,
    kind='reg',
    ci=95)
plt.show()

Построение модели

Выполним оценку параметров и анализ простой линейной регрессии (simple linear regression), используя библиотеку statsmodels (https://www.statsmodels.org/) и входящий в нее модуль линейной регрессии Linear Regression (https://www.statsmodels.org/stable/regression.html).

Данный модуль включает в себя классы, реализующие различные методы оценки параметров моделей линейной регрессии, в том числе:

  • класс OLS (https://www.statsmodels.org/stable/generated/statsmodels.regression.linear_model.OLS.html#statsmodels.regression.linear_model.OLS) — Ordinary Least Squares (обычный метод наименьших квадратов).

  • класс WLS (https://www.statsmodels.org/stable/generated/statsmodels.regression.linear_model.WLS.html#statsmodels.regression.linear_model.WLS) — Weighted Least Squares (метод взвешенных наименьших квадратов) (https://en.wikipedia.org/wiki/Weighted_least_squares), применяется, если имеет место гетероскедастичность данных (https://ru.wikipedia.org/wiki/Гетероскедастичность).

  • класс GLS (https://www.statsmodels.org/stable/generated/statsmodels.regression.linear_model.GLS.html#statsmodels.regression.linear_model.GLS) — Generalized Least Squares (обобщенный метод наименьших квадратов) (https://en.wikipedia.org/wiki/Generalized_least_squares), применяется, если существует определенная степень корреляции между остатками в модели регрессии.

  • класс GLSAR (https://www.statsmodels.org/stable/generated/statsmodels.regression.linear_model.GLSAR.html#statsmodels.regression.linear_model.GLSAR) — Generalized Least Squares with AR covariance structure (обобщенный метод наименьших квадратов, ковариационная структура с автокорреляцией — экспериментальный метод)

  • класс RecurciveLS (https://www.statsmodels.org/stable/examples/notebooks/generated/recursive_ls.html) — Recursive least squares (рекурсивный метод наименьших квадратов) (https://en.wikipedia.org/wiki/Recursive_least_squares_filter)

  • классы RollingOLS (https://www.statsmodels.org/stable/generated/statsmodels.regression.rolling.RollingOLS.html#statsmodels.regression.rolling.RollingOLS) и RollingWLS (https://www.statsmodels.org/stable/generated/statsmodels.regression.rolling.RollingWLS.html#statsmodels.regression.rolling.RollingWLS) — скользящая регрессия (https://www.statsmodels.org/stable/examples/notebooks/generated/rolling_ls.html, https://help.fsight.ru/ru/mergedProjects/lib/01_regression_models/rolling_regression.htm)

    и т.д.

Так как исходные данные подчиняются нормальному закону распределения и аномальные значения (выбросы) отсутствуют, воспользуемся для оценки параметров обычным методом наименьших квадратов (класс OLS):

model_linear_ols = smf.ols(formula='Y ~ X', data=dataset_df)
result_linear_ols = model_linear_ols.fit()
print(result_linear_ols.summary())

Альтернативная форма выдачи результатов:

print(result_linear_ols.summary2())

Результаты построения модели мы получаем как класс statsmodels.regression.linear_model.RegressionResults (https://www.statsmodels.org/stable/generated/statsmodels.regression.linear_model.RegressionResults.html#statsmodels.regression.linear_model.RegressionResults).

Экспресс-выводы, которые мы можем сразу сделать из результатов построения модели:

  1. Коэффициенты регрессии модели Y = b0 + b1∙X:

    • Intercept = b0 = -21.3741

    • b1 = 0.0129

  2. Коэффициент детерминации R-squared = 0.776, его скорректированная оценка Adj. R-squared = 0.757 — это означает, что регрессионная модуль объясняет 75.75% вариации переменной Y.

  3. Проверка значимости коэффициента детерминации:

    • расчетное значение статистики критерия Фишера: F-statistic = 41.61

    • расчетный уровень значимости Prob (F-statistic) = 3.16e-05

    • так как значение Prob (F-statistic) < 0.05, то нулевая гипотеза R-squared = 0 НЕ ПРИНИМАЕТСЯ, т.е. коэффициент детерминации ЗНАЧИМ

  4. Проверка значимости коэффициентов регрессии:

    • расчетный уровень значимости P>|t| не превышает 0.05 — это означает, что оба коэффициента регрессии значимы

    • об этом же свидетельствует то, что доверительный интервал для обоих коэффициентов регрессии ([0.025; 0.975]) не включает в себя точку 0

    Также в таблице результатов содержится прочая информация по коэффициентам регрессии: стандартная ошибка Std.Err. расчетное значение статистики критерия Стьюдента t для проверки гипотезы о значимости.

  5. Анализ остатков модели:

    • Тест Omnibus — про этот тест подробно написано в https://en.wikipedia.org/wiki/Omnibus_test, https://medium.com/swlh/interpreting-linear-regression-through-statsmodels-summary-4796d359035a, http://work.thaslwanter.at/Stats/html/statsModels.html.

      Расчетное значение статистики критерия Omnibus = 3.466 — по сути расчетное значение F-критерия (см. https://en.wikipedia.org/wiki/Omnibus_test).

      Prob(Omnibus) = 0.177 — показывает вероятность нормального распределения остатков (значение 1 указывает на совершенно нормальное распределение).

      Учитывая, что в дальнейшем мы проверим нормальность распределения остатков по совокупности различных тестов, в том числе с достаточно высокой мощностью, и все тесты позволят принять гипотезу о нормальном распределении — в данном случае к тесту Omnibus возникают вопросы. С этим тестом нужно разбираться отдельно.

    • Skew = 0.014 и Kurtosis = 1.587 — показатели асимметрии и эксцесса остатков свидетельствуют, что распределение остатков практически симметричное, островершинное.

    • проверка нормальности распределения остатков по критерию Харке-Бера: расчетное значение статистики критерия Jarque-Bera (JB) = 1.164 и расчетный уровень значимости Prob(JB) = 0.559. К данным результатам также возникают вопросы, особенно, если учесть, что критерий Харке-Бера является асимптотическим, расчетное значение имеет распределение хи-квадрат, поэтому данный критерий рекомендуют применять только для больших выборок (см. https://en.wikipedia.org/wiki/Jarque–Bera_test). Проверку нормальности распределения остатков модели лучше проводить с использованием набора стандартных статистических тестов python (см. далее).

  6. Проверка автокорреляции по критерию Дарбина-Уотсона: Durbin-Watson = 1.443.

    Мы не будем здесь разбирать данный критерий, так как явление автокорреляции больше характерно для данных, выражаемых в виде временных рядов. Однако, для грубой оценки считается, что при расчетном значении статистики криетрия Дарбина=Уотсона а интервале [1; 2] автокорреляция отсутствует (см.https://en.wikipedia.org/wiki/Durbin–Watson_statistic).

    Более подробно про критерий Дарбина-Уотсона — см. [1, с.659].

Прочая информация, которую можно извлечь из результатов построения модели:

  1. Covariance Type — тип ковариации, подробнее см. https://habr.com/ru/post/681218/, https://towardsdatascience.com/simple-explanation-of-statsmodel-linear-regression-model-summary-35961919868b#:~:text=Covariance type is typically nonrobust,with respect to each other.

  2. Scale — масштабный коэффициент для ковариационной матрицы (https://www.statsmodels.org/stable/generated/statsmodels.regression.linear_model.RegressionResults.scale.html#statsmodels.regression.linear_model.RegressionResults.scale), равен величине Mean squared error (MSE) (cреднеквадратической ошибке), об подробнее см. далее, в разделе про ошибки аппроксимации моделей.

  3. Показатели сравнения качества различных моделей:

    • Log-Likelihood — логарифмическая функция правдоподобия, подробнее см. https://en.wikipedia.org/wiki/Likelihood_function#Log-likelihood, https://habr.com/ru/post/433804/

    • AIC — информационный критерий Акаике (Akaike information criterion), подробнее см. https://en.wikipedia.org/wiki/Akaike_information_criterion

    • BIC — информационный критерий Байеса (Bayesian information criterion), подробнее см. https://en.wikipedia.org/wiki/Bayesian_information_criterion

    В данной статье мы эти показатели рассматривать не будем, так как задача выбора одной модели из нескольких перед нами не стоит.

  4. Число обусловленности Cond. No = 96792 используется для проверки мультиколлинеарности (считается, что мультиколлинеарность есть, если значение Cond. No > 30) (см. http://work.thaslwanter.at/Stats/html/statsModels.html). В нашем случае парной регрессионной модели о мультиколлинеарности речь не идет.

Далее будем извлекать данные из стандартного набора выдачи результатов и анализировать их более подробно. Последующие этапы вовсе не обязательно проводить в полном объеме при решении задач, но здесь мы рассмотрим их подробно.

Параметры и уравнение регрессионной модели

Извлечем параметры полученной модели — как свойство params модели:

print('Параметры модели: \n', result_linear_ols.params, type(result_linear_ols.params))

Имея параметры модели, можем формализовать уравнение модели Y = b0 + b1*X:

b0 = result_linear_ols.params['Intercept']
b1 = result_linear_ols.params['X']
Y_calc = lambda x: b0 + b1*x

График регрессионной модели

Для построения графиков регрессионных моделей можно воспользоваться стандартными возможностями библиотек statsmodels, seaborn, либо создать пользовательскую функцию — на усмотрение исследователя:

1. Построение графиков регрессионных моделей с использованием библиотеки statsmodels

С помощью функции statsmodels.graphics.plot_fit (https://www.statsmodels.org/stable/generated/statsmodels.graphics.regressionplots.plot_fit.html#statsmodels.graphics.regressionplots.plot_fit) — отображается график Y and Fitted vs.X (фактические и расчетные значения Y с доверительным интервалом для каждого значения Y):

fig, ax = plt.subplots(figsize=(297/INCH, 210/INCH))
fig = sm.graphics.plot_fit(
    result_linear_ols, 'X',
    vlines=True,    # это параметр отвечает за отображение доверительных интервалов для Y
    ax=ax)
ax.set_ylabel(Variable_Name_Y)
ax.set_xlabel(Variable_Name_X)
ax.set_title(Task_Project)
plt.show()

С помощью функции statsmodels.graphics.plot_regress_exog (https://www.statsmodels.org/stable/generated/statsmodels.graphics.regressionplots.plot_regress_exog.html#statsmodels.graphics.regressionplots.plot_regress_exog) — отображается область 2х2, которая содержит:

  • предыдущий график Y and Fitted vs.X;

  • график остатков Residuals versus X;

  • график Partial regression plot — график частичной регрессии, пытается показать эффект добавления другой переменной в модель, которая уже имеет одну или несколько независимых переменных (более подробно см. https://en.wikipedia.org/wiki/Partial_regression_plot);

  • график CCPR Plot (Component-Component plus Residual Plot) — еще один способ оценить влияние одной независимой переменной на переменную отклика, принимая во внимание влияние других независимых переменных (более подробно — см. https://towardsdatascience.com/calculating-price-elasticity-of-demand-statistical-modeling-with-python-6adb2fa7824d, https://www.kirenz.com/post/2021-11-14-linear-regression-diagnostics-in-python/linear-regression-diagnostics-in-python/).

fig = plt.figure(figsize=(297/INCH, 210/INCH))
sm.graphics.plot_regress_exog(result_linear_ols, 'X', fig=fig)
plt.show()

2. Построение графиков регрессионных моделей с использованием библиотеки seaborn

Воспользуемся модулем regplot библиотеки seaborn (https://seaborn.pydata.org/generated/seaborn.regplot.html). Данный модуль позволяет визуализировать различные виды регрессии:

  • линейную

  • полиномиальную

  • логистическую

  • взвешенную локальную регрессию (LOWESS — Locally Weighted Scatterplot Smoothing) (см. http://www.machinelearning.ru/wiki/index.php?title=Алгоритм_LOWESS, https://www.statsmodels.org/stable/generated/statsmodels.nonparametric.smoothers_lowess.lowess.html)

Более подробно про модуль regplot можно прочитать в статье: https://pyprog.pro/sns/sns_8_regression_models.html.

Есть более совершенный модуль lmplot (https://seaborn.pydata.org/generated/seaborn.lmplot.html), который объединяет в себе regplot и FacetGrid, но мы его здесь рассматривать не будем.

# создание рисунка (Figure) и области рисования (Axes)
fig = plt.figure(figsize=(297/INCH, 420/INCH/1.5))
ax1 = plt.subplot(2,1,1)
ax2 = plt.subplot(2,1,2)
# заголовок рисунка (Figure)
title_figure = Task_Project
fig.suptitle(title_figure, fontsize = 18)
# заголовок области рисования (Axes)
title_axes_1 = 'Линейная регрессионная модель'
ax1.set_title(title_axes_1, fontsize = 16)
# график регрессионной модели
order_mod = 1    # порядок модели
#label_legend_regr_model = 'фактические данные'
sns.regplot(
    #data=dataset_df,
    x=X, y=Y,
    #x_estimator=np.mean,
    order=order_mod,
    logistic=False,
    lowess=False,
    robust=False,
    logx=False,
    ci=95,
    scatter_kws={'s': 30, 'color': 'red'},
    line_kws={'color': 'blue'},
    #label=label_legend_regr_model,
    ax=ax1)
ax1.set_ylabel(Variable_Name_Y)
ax1.legend()
# график остатков
title_axes_2 = 'График остатков'
ax2.set_title(title_axes_2, fontsize = 16)
sns.residplot(
    #data=dataset_df,
    x=X, y=Y,
    order=order_mod,
    lowess=False,
    robust=False,
    scatter_kws={'s': 30, 'color': 'darkorange'},
    ax=ax2)
ax2.set_xlabel(Variable_Name_X)

plt.show()

3. Построение графиков регрессионных моделей с помощью пользовательской функции

# Пользовательская функция
graph_regression_plot_sns(
    X, Y,
    regression_model=Y_calc,
    Xmin=Xmin_graph, Xmax=Xmax_graph,
    Ymin=Ymin_graph, Ymax=Ymax_graph,
    title_figure=Task_Project,
    x_label=Variable_Name_X,
    y_label=Variable_Name_Y,
    label_legend_regr_model=f'линейная регрессия Y = {b0:.3f} + {b1:.4f}*X',
    s=80,
    file_name='graph/regression_plot_lin.png')

Статистический анализ регрессионной модели

1. Расчет ошибки аппроксимации (Error Metrics)

Ошибки аппроксимации (Error Metrics) позволяют получить общее представление о качестве модели, а также позволяют сравнивать между собой различные модели.

Создадим пользовательскую функцию, которая рассчитывает основные ошибки аппроксимации для заданной модели:

  • Mean squared error (MSE) или Mean squared deviation (MSD) — среднеквадратическая ошибка (https://en.wikipedia.org/wiki/Mean_squared_error):

  • Root mean square error (RMSE) или Root mean square deviation (RMSD) — квадратный корень из MSE (https://en.wikipedia.org/wiki/Root-mean-square_deviation):

  • Mean absolute error (MAE) — средняя абсолютная ошибка (https://en.wikipedia.org/wiki/Mean_absolute_error):

  • Mean squared prediction error (MSPE) — среднеквадратическая ошибка прогноза (среднеквадратическая ошибка в процентах) (https://en.wikipedia.org/wiki/Mean_squared_prediction_error):

  • Mean absolute percentage error (MAPE) — средняя абсолютная ошибка в процентах (https://en.wikipedia.org/wiki/Mean_absolute_percentage_error):

Про выбор метрики см. также https://machinelearningmastery.ru/how-to-select-the-right-evaluation-metric-for-machine-learning-models-part-2-regression-metrics-d4a1a9ba3d74/.

# Пользовательская функция
def regression_error_metrics(model, model_name=''):
    model_fit = model.fit()
    Ycalc = model_fit.predict()
    n_fit = model_fit.nobs
    Y = model.endog
    
    MSE = (1/n_fit) * np.sum((Y-Ycalc)**2)
    RMSE = sqrt(MSE)
    MAE = (1/n_fit) * np.sum(abs(Y-Ycalc))
    MSPE = (1/n_fit) *  np.sum(((Y-Ycalc)/Y)**2)
    MAPE = (1/n_fit) *  np.sum(abs((Y-Ycalc)/Y))
        
    model_error_metrics = {
        'MSE': MSE,
        'RMSE': RMSE,
        'MAE': MAE,
        'MSPE': MSPE,
        'MAPE': MAPE}
    
    result = pd.DataFrame({
        'MSE': MSE,
        'RMSE': RMSE,
        'MAE': MAE,
        'MSPE': "{:.3%}".format(MSPE),
        'MAPE': "{:.3%}".format(MAPE)},
        index=[model_name])        
        
    return model_error_metrics, result

(model_error_metrics, result) = regression_error_metrics(model_linear_ols, model_name='linear_ols')
display(result)

В литературе по прикладной статистике нет единого мнения о допустимом размере относительных ошибок аппроксимации: в одних источниках допустимой считается ошибка 5-7%, в других она может быть увеличена до 8-10%, и даже до 15%.

Вывод: модель хорошо аппроксимирует фактические данные (относительная ошибка аппроксимации MAPE = 3.405% < 10%).

2. Дисперсионный анализ регрессионной модели (ДАРМ)

ДАРМ не входит в стандартную форму выдачи результатов Regression Results, однако я решил написать здесь о нем по двум причинам:

  1. Именно анализ дисперсии регрессионной модели, разложение этой дисперсии на составляющие дает фундаментальное представление о сути регрессии, а термины, используемые при ДАРМ, применяются на последующих этапах анализа.

  2. С терминами ДАРМ в литературе по прикладной статистике имеется некоторая путаница, в разных источниках они могут именоваться по-разному (см., например, [8, с.52]), поэтому, чтобы двигаться дальше, необходимо определиться с понятиями.

При ДАРМ общую вариацию результативного признака (Y) принято разделять на две составляющие — вариация, обусловленная регрессией и вариация, обусловленная отклонениями от регрессии (остаток), при этом в разных источниках эти термины могут именоваться и обозначаться по-разному, например:

  1. Вариация, обусловленная регрессией — может называться Explained sum of squares (ESS), Sum of Squared Regression (SSR) (https://en.wikipedia.org/wiki/Explained_sum_of_squares, https://towardsdatascience.com/anova-for-regression-fdb49cf5d684), Sum of squared deviations (SSD).

  2. Вариация, обусловленная отклонениями от регрессии (остаток) — может называться Residual sum of squares (RSS), Sum of squared residuals (SSR), Squared estimate of errors, Sum of Squared Error (SSE) (https://en.wikipedia.org/wiki/Residual_sum_of_squares, https://towardsdatascience.com/anova-for-regression-fdb49cf5d684); в отчественной практике также применяется термин остаточная дисперсия.

  3. Общая (полная) вариация — может называться Total sum of squares (TSS), Sum of Squared Total (SST) (https://en.wikipedia.org/wiki/Partition_of_sums_of_squares, https://towardsdatascience.com/anova-for-regression-fdb49cf5d684).

Как видим, путаница знатная:

  • в разных источниках под SSR могут подразумеваться различные показатели;

  • легко перепутать показатели ESS и SSE;

  • в библиотеке statsmodel также есть смешение терминов: для показателя Explained sum of squares используется свойство ess, а для показателя Sum of squared (whitened) residuals — свойство ssr.

Мы будем пользоваться системой обозначений, принятой в большинстве источников — SSR (Sum of Squared Regression), SSE (Sum of Squared Error), SST (Sum of Squared Total). Стандартная таблица ДАРМ в этом случае имеет вид:

Примечания:

  1. Здесь приведена таблица ДАРМ для множественной линейной регрессионной модели (МЛРМ), в нашем случае при ПЛРМ мы имеем частный случай p=1.

  2. Показатели Fcalc-ad и Fcalc-det — расчетные значения статистики критерия Фишера при проверке адекватности модели и значимости коэффициента детерминации (об этом — см.далее).

Более подробно про дисперсионный анализ регрессионной модели — см.[4, глава 3].

Класс statsmodels.regression.linear_model.RegressionResults позволяет нам получить данные для ANOVA (см. https://www.statsmodels.org/stable/generated/statsmodels.regression.linear_model.RegressionResults.html#statsmodels.regression.linear_model.RegressionResults) как свойства класса:

  1. Сумма квадратов, обусловленная регрессией / SSR (Sum of Squared Regression) — свойство ess.

  2. Сумма квадратов, обусловленная отклонением от регрессии / SSE (Sum of Squared Error) — свойство ssr.

  3. Общая (полная) сумма квадратов / SST (Sum of Squared Total) — свойство centered_tss.

  4. Кол-во наблюдений / Number of observations — свойство nobs.

  5. Число степеней свободы модели / Model degrees of freedom — равно числу переменных модели (за исключением константы, если она присутствует — свойство df_model.

  6. Среднеквадратичная ошибка модели / Mean squared error the model — сумма квадратов, объясненная регрессией, деленная на число степеней свободы регрессии — свойство mse_model.

  7. Среднеквадратичная ошибка остатков / Mean squared error of the residuals — сумма квадратов остатков, деленная на остаточные степени свободы — свойство mse_resid.

  8. Общая среднеквадратичная ошибка / Total mean squared error — общая сумма квадратов, деленная на количество наблюдений — свойство mse_total.

Также имеется модуль statsmodels.stats.anova.anova_lm, который позволяет получить результаты ДАРМ (нескольких типов — 1, 2, 3):

# тип 1
print('The type of Anova test: 1')
display(sm.stats.anova_lm(result_linear_ols, typ=1))

# тип 2
print('The type of Anova test: 2')
display(sm.stats.anova_lm(result_linear_ols, typ=2))

# тип 3
print('The type of Anova test: 3')
display(sm.stats.anova_lm(result_linear_ols, typ=3))

На мой взгляд, форма таблица результатов statsmodels.stats.anova.anova_lm не вполне удобна, поэтому сформируем ее самостоятельно, для чего создадим пользовательскую функцию ANOVA_table_regression_model:

# Пользовательская функция
def ANOVA_table_regression_model(model_fit):
    n = int(model_fit.nobs)
    p = int(model_fit.df_model)
    SSR = model_fit.ess
    SSE = model_fit.ssr
    SST = model_fit.centered_tss

    result = pd.DataFrame({
        'sources_of_variation': ('regression (SSR)', 'deviation from regression (SSE)', 'total (SST)'),
        'sum_of_squares': (SSR, SSE, SST),
        'degrees_of_freedom': (p, n-p-1, n-1)})
    result['squared_error'] = result['sum_of_squares'] / result['degrees_of_freedom']
    R2 = 1 - result.loc[1, 'sum_of_squares'] / result.loc[2, 'sum_of_squares']
    F_calc_adequacy = result.loc[2, 'squared_error'] / result.loc[1, 'squared_error']
    F_calc_determ_check = result.loc[0, 'squared_error'] / result.loc[1, 'squared_error']
    result['F-ratio'] = (F_calc_determ_check, F_calc_adequacy, '')
    
    return result

result = ANOVA_table_regression_model(result_linear_ols)
display(result)
print(f"R2 = 1 - SSE/SST = {1 - result.loc[1, 'sum_of_squares'] / result.loc[2, 'sum_of_squares']}")
print(f"F_calc_adequacy = MST / MSE = {result.loc[2, 'squared_error'] / result.loc[1, 'squared_error']}")
print(f"F_calc_determ_check = MSR / MSE = {result.loc[0, 'squared_error'] / result.loc[1, 'squared_error']}")

ДАРМ позволяет визуализировать вариацию:

fig, axes = plt.subplots(figsize=(210/INCH, 297/INCH/1.5))
axes.pie(
    (result.loc[0, 'sum_of_squares'], result.loc[1, 'sum_of_squares']), 
    labels=(result.loc[0, 'sources_of_variation'], result.loc[1, 'sources_of_variation']),
    autopct='%.1f%%',
    startangle=60)
plt.show()

На основании данных ДАРМ мы рассчитали ряд показателей (R2, Fcalc-ad и Fcalc-det), которые будут использоваться в дальнейшем.

3. Анализ остатков (проверка нормальности распределения остатков и гипотезы о равенстве нулю среднего значения остатков)

Проверка нормальности распределения остатков — один их важнейших этапов анализа регрессионной модели. Требование нормальности распределения остатков не требуется для отыскания параметров модели, но необходимо в дальнейшем для проверки статистических гипотез с использованием критериев Фишера и Стьюдента (проверка адекватности модели, значимости коэффициента детерминации, значимости коэффициентов регрессии) и построения доверительных интервалов [5, с.122].

Остатки регрессионной модели:

print('Остатки регрессионной модели:\n', result_linear_ols.resid, type(result_linear_ols.resid))
res_Y = np.array(result_linear_ols.resid)

statsmodels может выдавать различные преобразованные виды остатков (см. https://www.statsmodels.org/stable/generated/statsmodels.regression.linear_model.RegressionResults.resid_pearson.html, https://www.statsmodels.org/stable/generated/statsmodels.regression.linear_model.RegressionResults.wresid.html).

График остатков:

# Пользовательская функция
graph_scatterplot_sns(
    X, res_Y,
    Xmin=Xmin_graph, Xmax=Xmax_graph,
    Ymin=-3.0, Ymax=3.0,
    color='red',
    #title_figure=Task_Project,
    title_axes='Остатки линейной регрессионной модели', title_axes_fontsize=18,
    x_label=Variable_Name_X,
    y_label='ΔY = Y - Ycalc',
    s=75,
    file_name='graph/residuals_plot_sns.png')

Проверка нормальности распределения остатков:

# Пользовательская функция
graph_hist_boxplot_probplot_sns(
    data=res_Y,
    data_min=-2.5, data_max=2.5,
    graph_inclusion='bp',
    data_label='ΔY = Y - Ycalc',
    #title_figure=Task_Project,
    title_axes='Остатки линейной регрессионной модели', title_axes_fontsize=16,
    file_name='graph/residuals_hist_boxplot_probplot_sns.png')    

norm_distr_check(res_Y)

Вывод: большинство статистических тестов позволяют принять гипотезу о нормальности распределения остатков.

Проверка гипотезы о равенстве нулю среднего значения остатков — так как остатки имеют нормальное распределение, воспользуемся критерием Стьюдента (функция scipy.stats.ttest_1samp, https://docs.scipy.org/doc/scipy/reference/generated/scipy.stats.ttest_ind.html):

sps.ttest_1samp(res_Y, popmean=0)

Вывод: так как расчетный уровень значимости превышает заданный (0.05), то нулевая гипотеза о равенстве нулю остатков ПРИНИМАЕТСЯ.

4. Проверка адекватности модели

Суть проверки адекватности регрессионной модели заключается в сравнении полной дисперсии MST и остаточной дисперсии MSE — проверяется гипотеза о равенстве этих дисперсий по критерию Фишера. Если дисперсии различаются значимо, то модель считается адекватной. Более подробно про проверку адекватности регрессионной — см.[1, с.658], [2, с.49], [4, с.154].

Для проверки адекватности регрессионной модели создадим пользовательскую функцию regression_model_adequacy_check:

def regression_model_adequacy_check(
    model_fit,
    p_level: float=0.95,
    model_name=''):
    
    n = int(model_fit.nobs)
    p = int(model_fit.df_model)    # Число степеней свободы регрессии, равно числу переменных модели (за исключением константы, если она присутствует)
    
    SST = model_fit.centered_tss    # SST (Sum of Squared Total)
    dfT = n-1
    MST = SST / dfT

    SSE = model_fit.ssr    # SSE (Sum of Squared Error)
    dfE = n - p - 1
    MSE = SSE / dfE
    
    F_calc = MST / MSE
    F_table = sci.stats.f.ppf(p_level, dfT, dfE, loc=0, scale=1)
    a_calc = 1 - sci.stats.f.cdf(F_calc, dfT, dfE, loc=0, scale=1)
    conclusion_model_adequacy_check = 'adequacy' if F_calc >= F_table else 'adequacy'
    
    # формируем результат            
    result = pd.DataFrame({
        'SST': (SST),
        'SSE': (SSE),
        'dfT': (dfT),
        'dfE': (dfE),
        'MST': (MST),
        'MSE': (MSE),
        'p_level': (p_level),
        'a_level': (a_level),
        'F_calc': (F_calc),
        'F_table': (F_table),
        'F_calc >= F_table': (F_calc >= F_table),
        'a_calc': (a_calc),
        'a_calc <= a_level': (a_calc <= a_level),
        'adequacy_check': (conclusion_model_adequacy_check),
        },
        index=[model_name]
        )
    
    return result

regression_model_adequacy_check(result_linear_ols, p_level=0.95, model_name='linear_ols')

Вывод: модель является АДЕКВАТНОЙ.

5. Коэффициент детерминации и проверка его значимости

Различают несколько видов коэффициента детерминации:

  1. Собственно обычный коэффициент детерминации:

Его значение может быть получено как свойство rsquared модели.

  1. Скорректированный (adjusted) коэффициент детерминации — используется для того, чтобы была возможность сравнивать модели с разным числом признаков так, чтобы число регрессоров (признаков) не влияло на статистику R2, при его расчете используются несмещённые оценки дисперсий:

Его значение может быть получено как свойство rsquared_adj модели.

  1. Обобщённый (extended) коэффициент детерминации — используется для сравнения моделей регрессии со свободным членом и без него, а также для сравнения между собой регрессий, построенных с помощью различных методов: МНК, обобщённого метода наименьших квадратов (ОМНК), условного метода наименьших квадратов (УМНК), обобщённо-условного метода наименьших квадратов (ОУМНК). В данном разборе ПЛРМ рассматривать этот коэффициент мы не будем.

Более подробно с теорией вопроса можно ознакомиться, например: http://www.machinelearning.ru/wiki/index.php?title=Коэффициент_детерминации), а также в [7].

Значения коэффициента детерминации и скорректированного коэффициента детерминации, извлеченные с помощью свойств rsquared и rsquared_adj модели.

print('R2 =', result_linear_ols.rsquared)
print('R2_adj =', result_linear_ols.rsquared_adj)

Значимость коэффициента детерминации можно проверить по критерию Фишера [3, с.201-203; 8, с.83].

Расчетное значение статистики критерия Фишера может быть получено с помощью свойства fvalue модели:

print(f"result_linear_ols.fvalue = {result_linear_ols.fvalue}")

Расчетный уровень значимости при проверке гипотезы по критерию Фишера может быть получено с помощью свойства f_pvalue модели:

print(f"result_linear_ols.f_pvalue = {result_linear_ols.f_pvalue}")

Можно рассчитать уровень значимости самостоятельно (так сказать, для лучшего понимания и общей демонстрации возможностей) — для этого воспользуемся библиотекой scipy, модулем распределения Фишера scipy.stats.f, свойством cdf (функция распределения):

df1 = int(result_linear_ols.df_model)
df2 = int(result_linear_ols.nobs - result_linear_ols.df_model - 1)
F_calc = result_linear_ols.fvalue
a_calc = 1 - sci.stats.f.cdf(F_calc, df1, df2, loc=0, scale=1)
print(a_calc)

Как видим, результаты совпадают.

Табличное значение статистики критерия Фишера получить с помощью Regression Results нельзя. Рассчитаем его самостоятельно — для этого воспользуемся библиотекой scipy, модулем распределения Стьюдента scipy.stats.f, свойством ppf (процентные точки):

F_table = sci.stats.f.ppf(p_level, df1, df2, loc=0, scale=1)
print(F_table)

Для удобства создадим пользовательскую функцию determination_coef_check для проверки значимости коэффициента детерминации, которая объединяет все вышеперечисленные расчеты:

# Пользовательская функция
def determination_coef_check(
    model_fit,
    p_level: float=0.95):
    
    a_level = 1 - p_level
    
    R2 = model_fit.rsquared
    R2_adj = model_fit.rsquared_adj
    n = model_fit.nobs    # объем выборки
    p = model_fit.df_model    # Model degrees of freedom. The number of regressors p. Does not include the constant if one is present.
    
    F_calc = R2 / (1 - R2) * (n-p-1)/p
    df1 = int(p)
    df2 = int(n-p-1)
    F_table = sci.stats.f.ppf(p_level, df1, df2, loc=0, scale=1)
    a_calc = 1 - sci.stats.f.cdf(F_calc, df1, df2, loc=0, scale=1)
    conclusion_determ_coef_sign = 'significance' if F_calc >= F_table else 'not significance'
        
    # формируем результат            
    result = pd.DataFrame({
        'notation': ('R2'),
        'coef_value (R)': (sqrt(R2)),
        'coef_value_squared (R2)': (R2),
        'p_level': (p_level),
        'a_level': (a_level),
        'F_calc': (F_calc),
        'df1': (df1),
        'df2': (df2),
        'F_table': (F_table),
        'F_calc >= F_table': (F_calc >= F_table),
        'a_calc': (a_calc),
        'a_calc <= a_level': (a_calc <= a_level),
        'significance_check': (conclusion_determ_coef_sign),
        'conf_int_low': (''),
        'conf_int_high': ('')
        },
        index=['Coef. of determination'])
    return result

determination_coef_check(
    result_linear_ols,
    p_level=0.95)

Вывод: коэффициент детерминации ЗНАЧИМ.

6. Коэффициенты регрессии и проверка их значимости

Ранее мы уже извлекли коэффициенты регрессии как параметры модели b0 и b1 (как свойство params модели). Также можно получить их значения, как свойство bse модели:

print(b0, b1)
print(result_linear_ols.bse, type(result_linear_ols.bse))

Значимость коэффициентов регрессии можно проверить по критерию Стьюдента [3, с.203-212; 8, с.78].

Расчетные значения статистики критерия Стьюдента могут быть получены с помощью свойства tvalues модели:

print(f"result_linear_ols.tvalues = \n{result_linear_ols.tvalues}")

Расчетные значения уровня значимости при проверке гипотезы по критерию Стьюдента могут быть получены с помощью свойства pvalues модели:

print(f"result_linear_ols.pvalues = \n{result_linear_ols.pvalues}")

Доверительные интервалы для коэффициентов регрессии могут быть получены с помощью свойства conf_int модели:

print(result_linear_ols.conf_int(), '\n')

Можно рассчитать уровень значимости самостоятельно (как ранее для критерия Фишера — для лучшего понимания и общей демонстрации возможностей) — для этого воспользуемся библиотекой scipy, модулем распределения Стьюдента scipy.stats.t, свойством cdf (функция распределения):

t_calc = result_linear_ols.tvalues
df = int(result_linear_ols.nobs - result_linear_ols.df_model - 1)
a_calc = 2*(1-sci.stats.t.cdf(abs(t_calc), df, loc=0, scale=1))
print(a_calc)

Как видим, результаты совпадают.

Табличные значения статистики критерия Стьюдента получить с помощью Regression Results нельзя. Рассчитаем их самостоятельно — для этого воспользуемся библиотекой scipy, модулем распределения Стьюдента scipy.stats.t, свойством ppf (процентные точки):

t_table = sci.stats.t.ppf((1 + p_level)/2 , df)
print(t_table)

Для удобства создадим пользовательскую функцию regression_coef_check для проверки значимости коэффициентов регрессии, которая объединяет все вышеперечисленные расчеты:

def regression_coef_check(
    model_fit,
    notation_coef: list='',
    p_level: float=0.95):
    
    a_level = 1 - p_level
    
    # параметры модели (коэффициенты регрессии)
    model_params = model_fit.params
    # стандартные ошибки коэффициентов регрессии
    model_bse = model_fit.bse
    # проверка гипотезы о значимости регрессии
    t_calc = abs(model_params) / model_bse
    n = model_fit.nobs    # объем выборки
    p = model_fit.df_model    # Model degrees of freedom. The number of regressors p. Does not include the constant if one is present.
    df = int(n - p - 1)
    t_table = sci.stats.t.ppf((1 + p_level)/2 , df)
    a_calc = 2*(1-sci.stats.t.cdf(t_calc, df, loc=0, scale=1))
    conclusion_ = ['significance' if elem else 'not significance' for elem in (t_calc >= t_table).values]
        
    # доверительный интервал коэффициента регрессии
    conf_int_low = model_params - t_table*model_bse
    conf_int_high = model_params + t_table*model_bse
    
    # формируем результат            
    result = pd.DataFrame({
        'notation': (notation_coef),
        'coef_value': (model_params),
        'std_err': (model_bse),
        'p_level': (p_level),
        'a_level': (a_level),
        't_calc': (t_calc),
        'df': (df),
        't_table': (t_table),
        't_calc >= t_table': (t_calc >= t_table),
        'a_calc': (a_calc),
        'a_calc <= a_level': (a_calc <= a_level),
        'significance_check': (conclusion_),
        'conf_int_low': (conf_int_low),
        'conf_int_high': (conf_int_high),
        })
    
    return result

regression_coef_check(
    result_linear_ols,
    notation_coef=['b0', 'b1'],
    p_level=0.95)

Вывод: коэффициенты регрессии b0 и b1 ЗНАЧИМЫ.

7. Проверка гетероскедастичности

Для проверка гетероскедастичности statsmodels предлагает нам следующие инструменты:

  • тест Голдфелда-Квандта (https://www.statsmodels.org/stable/generated/statsmodels.stats.diagnostic.het_goldfeldquandt.html#statsmodels.stats.diagnostic.het_goldfeldquandt) — теорию см. [8, с.178], также https://ru.wikipedia.org/wiki/Тест_Голдфелда_—_Куандта.

  • тест Бриша-Пэгана (Breush-Pagan test) (https://www.statsmodels.org/stable/generated/statsmodels.stats.diagnostic.het_breuschpagan.html#statsmodels.stats.diagnostic.het_breuschpagan) — теорию см.[8, с.179], также https://en.wikipedia.org/wiki/Breusch–Pagan_test.

  • тест Уайта (White test) (https://www.statsmodels.org/stable/generated/statsmodels.stats.diagnostic.het_white.html#statsmodels.stats.diagnostic.het_white) — теорию см.[8, с.177], а также https://ru.wikipedia.org/wiki/Тест_Уайта.

    Тест Голдфелда-Квандта (Goldfeld–Quandt test)

# тест Голдфелда-Квандта (Goldfeld–Quandt test)
test = sms.het_goldfeldquandt(result_linear_ols.resid, result_linear_ols.model.exog)
test_result = lzip(['F_calc', 'p_calc'], test)    # распаковка результатов теста
# расчетное значение статистики F-критерия
F_calc_tuple = test_result[0]
F_calc = F_calc_tuple[1]
print(f"Расчетное значение статистики F-критерия: F_calc = {round(F_calc, DecPlace)}")
# расчетный уровень значимости
p_calc_tuple = test_result[1]
p_calc = p_calc_tuple[1]
print(f"Расчетное значение доверительной вероятности: p_calc = {round(p_calc, DecPlace)}")
#a_calc = 1 - p_value
#print(f"Расчетное значение уровня значимости: a_calc = 1 - p_value = {round(a_calc, DecPlace)}")
# вывод
if p_calc < a_level:
    conclusion_GQ_test = f"Так как p_calc = {round(p_calc, DecPlace)} < a_level = {round(a_level, DecPlace)}" + \
        ", то дисперсии в подвыборках отличаются значимо, т.е. гипотеза о наличии гетероскедастичности ПРИНИМАЕТСЯ"
else:
    conclusion_GQ_test = f"Так как p_calc = {round(p_calc, DecPlace)} >= a_level = {round(a_level, DecPlace)}" + \
        ", то дисперсии в подвыборках отличаются незначимо, т.е. гипотеза о наличии гетероскедастичности ОТВЕРГАЕТСЯ"
print(conclusion_GQ_test)

Для удобства создадим пользовательскую функцию Goldfeld_Quandt_test:

def Goldfeld_Quandt_test(
    model_fit,
    p_level: float=0.95,
    model_name=''):
    
    a_level = 1 - p_level
    
    # реализация теста
    test = sms.het_goldfeldquandt(model_fit.resid, model_fit.model.exog)
    test_result = lzip(['F_statistic', 'p_calc'], test)    # распаковка результатов теста
    # расчетное значение статистики F-критерия
    F_calc_tuple = test_result[0]
    F_statistic = F_calc_tuple[1]
    # расчетный уровень значимости
    p_calc_tuple = test_result[1]
    p_calc = p_calc_tuple[1]
    # вывод
    conclusion_test = 'heteroscedasticity' if p_calc < a_level else 'not heteroscedasticity'
    
    result = pd.DataFrame({
        'test': ('Goldfeld–Quandt test'),
        'p_level': (p_level),
        'a_level': (a_level),
        'F_statistic': (F_statistic),
        'p_calc': (p_calc),
        'p_calc < a_level': (p_calc < a_level),
        'heteroscedasticity_check': (conclusion_test)
        },
        index=[model_name])
    
    return result

Goldfeld_Quandt_test(result_linear_ols, p_level=0.95, model_name='linear_ols')

Тест Бриша-Пэгана (Breush-Pagan test)

# тест Бриша-Пэгана (Breush-Pagan test)
name = ["Lagrange multiplier statistic", "p-value", "f-value", "f p-value"]
test = sms.het_breuschpagan(result_linear_ols.resid, result_linear_ols.model.exog)
lzip(name, test)

Для удобства создадим пользовательскую функцию Breush_Pagan_test:

def Breush_Pagan_test(
    model_fit,
    p_level: float=0.95,
    model_name=''):
    
    a_level = 1 - p_level
    
    # реализация теста
    test = sms.het_breuschpagan(model_fit.resid, model_fit.model.exog)
    name = ['Lagrange_multiplier_statistic', 'p_calc_LM', 'F_statistic', 'p_calc']
    test_result = lzip(name, test)    # распаковка результатов теста
    # расчетное значение статистики теста множителей Лагранжа
    LM_calc_tuple = test_result[0]
    Lagrange_multiplier_statistic = LM_calc_tuple[1]
    # расчетный уровень значимости статистики теста множителей Лагранжа
    p_calc_LM_tuple = test_result[1]
    p_calc_LM = p_calc_LM_tuple[1]
    # расчетное значение F-статистики гипотезы о том, что дисперсия ошибки не зависит от x
    F_calc_tuple = test_result[2]
    F_statistic = F_calc_tuple[1]
    # расчетный уровень значимости F-статистики
    p_calc_tuple = test_result[3]
    p_calc = p_calc_tuple[1]
    # вывод
    conclusion_test = 'heteroscedasticity' if p_calc < a_level else 'not heteroscedasticity'

    # вывод
    conclusion_test = 'heteroscedasticity' if p_calc < a_level else 'not heteroscedasticity'
    
    result = pd.DataFrame({
        'test': ('Breush-Pagan test'),
        'p_level': (p_level),
        'a_level': (a_level),
        'Lagrange_multiplier_statistic': (Lagrange_multiplier_statistic),
        'p_calc_LM': (p_calc_LM),
        'p_calc_LM < a_level': (p_calc_LM < a_level),
        'F_statistic': (F_statistic),
        'p_calc': (p_calc),
        'p_calc < a_level': (p_calc < a_level),
        'heteroscedasticity_check': (conclusion_test)
        },
        index=[model_name])
    
    return result

Breush_Pagan_test(result_linear_ols, p_level=0.95, model_name='linear_ols')

Тест Уайта (White test)

# тест Уайта (White test)
name = ["Lagrange multiplier statistic", "p-value", "f-value", "f p-value"]
test = sms.het_white(result_linear_ols.resid, result_linear_ols.model.exog)
lzip(name, test)

Для удобства создадим пользовательскую функцию White_test:

def White_test(
    model_fit,
    p_level: float=0.95,
    model_name=''):
    
    a_level = 1 - p_level
    
    # реализация теста
    test = sms.het_white(model_fit.resid, model_fit.model.exog)
    name = ['Lagrange_multiplier_statistic', 'p_calc_LM', 'F_statistic', 'p_calc']
    test_result = lzip(name, test)    # распаковка результатов теста
    # расчетное значение статистики теста множителей Лагранжа
    LM_calc_tuple = test_result[0]
    Lagrange_multiplier_statistic = LM_calc_tuple[1]
    # расчетный уровень значимости статистики теста множителей Лагранжа
    p_calc_LM_tuple = test_result[1]
    p_calc_LM = p_calc_LM_tuple[1]
    # расчетное значение F-статистики гипотезы о том, что дисперсия ошибки не зависит от x
    F_calc_tuple = test_result[2]
    F_statistic = F_calc_tuple[1]
    # расчетный уровень значимости F-статистики
    p_calc_tuple = test_result[3]
    p_calc = p_calc_tuple[1]
    # вывод
    conclusion_test = 'heteroscedasticity' if p_calc < a_level else 'not heteroscedasticity'

    # вывод
    conclusion_test = 'heteroscedasticity' if p_calc < a_level else 'not heteroscedasticity'
    
    result = pd.DataFrame({
        'test': ('White test'),
        'p_level': (p_level),
        'a_level': (a_level),
        'Lagrange_multiplier_statistic': (Lagrange_multiplier_statistic),
        'p_calc_LM': (p_calc_LM),
        'p_calc_LM < a_level': (p_calc_LM < a_level),
        'F_statistic': (F_statistic),
        'p_calc': (p_calc),
        'p_calc < a_level': (p_calc < a_level),
        'heteroscedasticity_check': (conclusion_test)
        },
        index=[model_name])
    
    return result

White_test(result_linear_ols, p_level=0.95, model_name='linear_ols')

Объединим результаты всех тестов гетероскедастичность в один DataFrame:

Goldfeld_Quandt_test_df = Goldfeld_Quandt_test(result_linear_ols, p_level=0.95, model_name='linear_ols')
Breush_Pagan_test_df = Breush_Pagan_test(result_linear_ols, p_level=0.95, model_name='linear_ols')
White_test_df = White_test(result_linear_ols, p_level=0.95, model_name='linear_ols')

heteroscedasticity_tests_df = pd.concat([Breush_Pagan_test_df, White_test_df, Goldfeld_Quandt_test_df])
display(heteroscedasticity_tests_df)

Выводы

Итак, мы провели статистический анализ регрессионной модели и установили:

  • исходные данные имеют нормальное распределение;

  • между переменными имеется весьма сильная корреляционная связь;

  • регрессионная модель хорошо аппроксимирует фактические данные;

  • остатки модели имеют нормальное распределение;

  • регрессионная модель адекватна по критерию Фишера;

  • коэффициент детерминации значим по критеию Фишера;

  • коэффициенты регрессии значимы по критерию Стьюдента;

  • гетероскедастичность отсутствует.

Применительно к рассматриваемой задаче выполнять проверку автокорреляции не имеет особого смысла из-за особенностей исходных данных (результаты замеров прочности бетона на разных участках здания).

Про статистический анализ регрессионных моделей с помощью statsmodels— см. еще https://www.statsmodels.org/stable/examples/notebooks/generated/regression_diagnostics.html.

Доверительные интервалы регрессионной модели

Для регрессионных моделей определяют доверительные интервалы двух видов [3, с.184-192; 4, с.172; 8, с.205-209]:

  1. Доверительный интервал средних значений переменной Y.

  2. Доверительный интервал индивидуальных значений переменной Y.

При этом размер доверительного интервала для индивидуальных значений больше, чем для средних значений.

Доверительные интервалы регрессионных моделей (ДИРМ) могут быть найдены разными способами:

  • непосредственно путем расчетов по формулам (см., например, https://habr.com/ru/post/558158/);

  • с использованием инструментария библиотеки statsmodels (см., например, https://www.stackfinder.ru/questions/17559408/confidence-and-prediction-intervals-with-statsmodels).

Разбререм более подробно способ с использованием библиотеки statsmodels. Прежде всего, с помощью свойства summary_table класса statsmodels.stats.outliers_influence.OLSInfluence (https://www.statsmodels.org/stable/generated/statsmodels.stats.outliers_influence.OLSInfluence.html?highlight=olsinfluence) мы можем получить таблицу данных, содержащую необходимую нам информацию:

  • Dep Var Population — фактические значения переменной Y;

  • Predicted Value — предсказанные значения переменной Y по по регрессионной модели;

  • Std Error Mean Predict — среднеквадратическая ошибка предсказанного среднего;

  • Mean ci 95% low и Mean ci 95% upp — границы доверительного интервала средних значений переменной Y;

  • Predict ci 95% low и Predict ci 95% upp — границы доверительного интервала индивидуальных значений переменной Y;

  • Residual — остатки регрессионной модели;

  • Std Error Residual — среднеквадратическая ошибка остатков;

  • Student Residual — стьюдентизированные остатки (подробнее см. http://statistica.ru/glossary/general/studentizirovannie-ostatki/);

  • Cook’s D — Расстояние Кука (Cook’s distance) — оценивает эффект от удаления одного (рассматриваемого) наблюдения; наблюдение считается выбросом, если Di > 4/n (более подробно — см.https://translated.turbopages.org/proxy_u/en-ru.ru.f584ceb5-63296427-aded8f31-74722d776562/https/en.wikipedia.org/wiki/Cook’s_distance, http://www.machinelearning.ru/wiki/index.php?title=Расстояние_Кука).

from statsmodels.stats.outliers_influence import summary_table
st, data, ss2 = summary_table(result_linear_ols, alpha=0.05)
print(st, '\n', type(st))

В нашем случае критическое значение расстояния Кука равно:

print(f'D_crit = 4/n = {4/result_linear_ols.nobs}')

то есть выбросов, смещающих оценки коэффициентов регрессии, не наблюдается.

Мы получили данные как класс statsmodels.iolib.table.SimpleTable. Свойство data преобразует данные в список. Далее для удобства работы преобразуем данные в DataFrame:

  st_data_df = pd.DataFrame(st.data)

Будем использовать данный DataFrame в дальнейшем, несколько преобразуем его:

  • изменим наименование столбцов (с цифр на названия показателей из таблицы summary_table)

  • удалим строки с текстовыми значениями

  • изменим индекс

  • добавим новый столбец — значения переменной X

  • отсортируем по возрастанию значений переменной X (это необходимо, чтобы графики границ доверительных интервалов выглядели как линии)

st_df = st_data_df.copy()
# изменим наименования столбцов
str = st_df.iloc[0,0:] + ' ' + st_df.iloc[1,0:]
st_df = st_df.rename(str, axis='columns')
# удалим строки 0, 1
st_df = st_df.drop([0,1])
# изменим индекс
st_df = st_df.set_index(np.arange(0, result_linear_ols.nobs))
# добавим новый столбец - значения переменной X
st_df.insert(1, 'X', X)
# отсортируем по возрастанию значений переменной X
st_df = st_df.sort_values(by='X')

display(st_df)

С помощью полученных данных мы можем построить график регрессионной модели с доверительными интервалами:

# создание рисунка (Figure) и области рисования (Axes)
fig, axes = plt.subplots(figsize=(297/INCH, 210/INCH))
# заголовок рисунка (Figure)
title_figure = Task_Project
fig.suptitle(title_figure, fontsize = 16)
# заголовок области рисования (Axes)
title_axes = 'Линейная регрессионная модель'
axes.set_title(title_axes, fontsize = 14)
# фактические данные
sns.scatterplot(
    x=st_df['X'], y=st_df['Dep Var Population'],
    label='фактические данные',
    s=50,
    color='red',
    ax=axes)
# график регрессионной модели
label_legend_regr_model=f'линейная регрессия Y = {b0:.3f} + {b1:.4f}*X'
sns.lineplot(
    x=st_df['X'], y=st_df['Predicted Value'],
    label=label_legend_regr_model,
    color='blue',
    ax=axes)
# доверительный интервал средних значений переменной Y
Mean_ci_low = st_df['Mean ci 95% low']
plt.plot(
    st_df['X'], Mean_ci_low,
    color='magenta', linestyle='--', linewidth=1,
    label='доверительный интервал средних значений Y')
Mean_ci_upp = st_df['Mean ci 95% upp']
plt.plot(
    st_df['X'], Mean_ci_upp,
    color='magenta', linestyle='--', linewidth=1)
# доверительный интервал индивидуальных значений переменной Y
Predict_ci_low = st_df['Predict ci 95% low']
plt.plot(
    st_df['X'], Predict_ci_low,
    color='orange', linestyle='-.', linewidth=2,
    label='доверительный интервал индивидуальных значений Y')
Predict_ci_upp = st_df['Predict ci 95% upp']
plt.plot(
    st_df['X'], Predict_ci_upp,
    color='orange', linestyle='-.', linewidth=2)

axes.set_xlabel(Variable_Name_X)
axes.set_ylabel(Variable_Name_Y)
axes.legend(prop={'size': 12})
plt.show()

Однако, мы получили данные о границах доверительных интервалов регрессионной модели только в пределах области фактических значений переменной X. Как быть, если мы хотим распространить прогноз за пределы этой области?

Прогнозирование

Под прогнозированием мы в данном случае будем понимать определение значений переменной Y и доверительных интервалов для ее средних и индивидуальных значений при заданном X. По сути, нам предстоит построить аналог рассмотренной выше таблицы summary_table, только с другими значениями X, причем эти значения могут выходить за пределы тех значений, которые использовались нами для построения регрессии.

Методика расчета доверительных интервалов регрессионных моделей разобрана в статье «Python, корреляция и регрессия: часть 4» (https://habr.com/ru/post/558158/), всем рекомендую ознакомиться.

Найти прогнозные значения Y не представляет труда, так как ранее мы уже формализовали модель в виде лямбда-функции, а вот для построения доверительных интервалов придется выполнить расчеты по формулам. Для этого создадим пользовательскую функцию regression_pair_predict, которая в случае парной регрессии (pair regression) для заданного значения X возвращает:

  • прогнозируемое по регрессионной модели значение y_calc

  • доверительный интервал [y_calc_mean_ci_low, y_calc_mean_ci_upp] средних значений переменной Y

  • доверительный интервал [y_calc_predict_ci_low, y_calc_predict_ci_upp] индивидуальных значений переменной Y

Алгоритм расчета доверительных интервалов для множественной регрессии (multiple regression) отличается и в данном обзоре не рассматривается (рассмотрим в дальнейшем).

Про прогнозирование с помощью регрессионных моделей — см.также:

  • https://www.statsmodels.org/stable/generated/statsmodels.regression.linear_model.RegressionResults.predict.html?highlight=predict#statsmodels.regression.linear_model.RegressionResults.predict

  • How to Make Predictions Using Regression Model in Statsmodels

  • https://www.statsmodels.org/stable/examples/notebooks/generated/predict.html

def regression_pair_predict(
    x_in,
    model_fit,
    regression_model,
    p_level: float=0.95):
    
    a_level = 1 - p_level
    
    X = pd.DataFrame(model_fit.model.exog)[1].values    # найти лучшее решение
    Y = model_fit.model.endog
    
    # вспомогательные величины
    n = int(result_linear_ols.nobs)
    SSE = model_fit.ssr    # SSE (Sum of Squared Error)
    dfE = n - p - 1
    MSE = SSE / dfE    # остаточная дисперсия
    
    Xmean = np.mean(X)
    SST_X = np.sum([(X[i] - Xmean)**2 for i in range(0, n)])
    
    t_table = sci.stats.t.ppf((1 + p_level)/2 , dfE)
    S2_y_calc_mean = MSE * (1/n + (x_in - Xmean)**2 / SST_X)
    S2_y_calc_predict = MSE * (1 + 1/n + (x_in - Xmean)**2 / SST_X)
        
    # прогнозируемое значение переменной Y
    y_calc=regression_model(x_in)
    # доверительный интервал средних значений переменной Y
    y_calc_mean_ci_low = y_calc - t_table*sqrt(S2_y_calc_mean)
    y_calc_mean_ci_upp = y_calc + t_table*sqrt(S2_y_calc_mean)
    # доверительный интервал индивидуальных значений переменной Y
    y_calc_predict_ci_low = y_calc - t_table*sqrt(S2_y_calc_predict)
    y_calc_predict_ci_upp = y_calc + t_table*sqrt(S2_y_calc_predict)
    
    result = y_calc, y_calc_mean_ci_low, y_calc_mean_ci_upp, y_calc_predict_ci_low, y_calc_predict_ci_upp
    
    return result

Сравним результаты расчета доверительных интервалов разными способами — с использованием функции regression_pair_predict и средствами statsmodels, для этого сформируем DaraFrame с новыми данными:

regression_pair_predict_df = pd.DataFrame(
    [regression_pair_predict(elem, result_linear_ols, regression_model=Y_calc) for elem in st_df['X'].values],
    columns=['y_calc', 'y_calc_mean_ci_low', 'y_calc_mean_ci_upp', 'y_calc_predict_ci_low', 'y_calc_predict_ci_upp'])
regression_pair_predict_df.insert(0, 'X', st_df['X'].values)
display(regression_pair_predict_df)

Видим, что результаты расчетов идентичны, следовательно мы можем использовать функцию regression_pair_predict для прогнозирования.

def graph_regression_pair_predict_plot_sns(
    model_fit,
    regression_model_in,
    Xmin=None, Xmax=None, Nx=10,
    Ymin_graph=None, Ymax_graph=None,
    title_figure=None, title_figure_fontsize=18,
    title_axes=None, title_axes_fontsize=16,
    x_label=None,
    y_label=None,
    label_fontsize=14, tick_fontsize=12, 
    label_legend_regr_model='', label_legend_fontsize=12,
    s=50, linewidth_regr_model=2,
    graph_size=(297/INCH, 210/INCH),
    result_output=True,
    file_name=None):
    
    # фактические данные
    X = pd.DataFrame(model_fit.model.exog)[1].values    # найти лучшее решение
    Y = model_fit.model.endog
    X = np.array(X)
    Y = np.array(Y)
    
    # границы
    if not(Xmin) and not(Xmax):
        Xmin=min(X)
        Xmax=max(X)
        Xmin_graph=min(X)*0.99
        Xmax_graph=max(X)*1.01
    else:
        Xmin_graph=Xmin
        Xmax_graph=Xmax
    
    if not(Ymin_graph) and not(Ymax_graph):
        Ymin_graph=min(Y)*0.99
        Ymax_graph=max(Y)*1.01       
    
    # формируем DataFrame данных
    Xcalc = np.linspace(Xmin, Xmax, num=Nx)
    Ycalc = regression_model_in(Xcalc)
    
    result_df = pd.DataFrame(
        [regression_pair_predict(elem, model_fit, regression_model=regression_model_in) for elem in Xcalc],
        columns=['y_calc', 'y_calc_mean_ci_low', 'y_calc_mean_ci_upp', 'y_calc_predict_ci_low', 'y_calc_predict_ci_upp'])
    result_df.insert(0, 'x_calc', Xcalc)
            
    # заголовки графика
    fig, axes = plt.subplots(figsize=graph_size)
    fig.suptitle(title_figure, fontsize = title_figure_fontsize)
    axes.set_title(title_axes, fontsize = title_axes_fontsize)
    
    # фактические данные
    sns.scatterplot(
        x=X, y=Y,
        label='фактические данные',
        s=s,
        color='red',
        ax=axes)
    
    # график регрессионной модели
    sns.lineplot(
        x=Xcalc, y=Ycalc,
        color='blue',
        linewidth=linewidth_regr_model,
        legend=True,
        label=label_legend_regr_model,
        ax=axes)
    
    # доверительный интервал средних значений переменной Y
    Mean_ci_low = result_df['y_calc_mean_ci_low']
    plt.plot(
        result_df['x_calc'], Mean_ci_low,
        color='magenta', linestyle='--', linewidth=1,
        label='доверительный интервал средних значений Y')
    
    Mean_ci_upp = result_df['y_calc_mean_ci_upp']
    plt.plot(
        result_df['x_calc'], Mean_ci_upp,
        color='magenta', linestyle='--', linewidth=1)
    
    # доверительный интервал индивидуальных значений переменной Y
    Predict_ci_low = result_df['y_calc_predict_ci_low']
    plt.plot(
        result_df['x_calc'], Predict_ci_low,
        color='orange', linestyle='-.', linewidth=2,
        label='доверительный интервал индивидуальных значений Y')
    
    Predict_ci_upp = result_df['y_calc_predict_ci_upp']
    plt.plot(
        result_df['x_calc'], Predict_ci_upp,
        color='orange', linestyle='-.', linewidth=2)
    
        
    axes.set_xlim(Xmin_graph, Xmax_graph)
    axes.set_ylim(Ymin_graph, Ymax_graph)        
    axes.set_xlabel(x_label, fontsize = label_fontsize)
    axes.set_ylabel(y_label, fontsize = label_fontsize)
    axes.tick_params(labelsize = tick_fontsize)
    #axes.tick_params(labelsize = tick_fontsize)
    axes.legend(prop={'size': label_legend_fontsize})
        
    plt.show()
    if file_name:
        fig.savefig(file_name, orientation = "portrait", dpi = 300)
        
    if result_output:
        return result_df
    else:
        return

graph_regression_pair_predict_plot_sns(
    model_fit=result_linear_ols,
    regression_model_in=Y_calc,
    Xmin=Xmin_graph-300, Xmax=Xmax_graph+200, Nx=25,
    Ymin_graph=Ymin_graph-5, Ymax_graph=Ymax_graph+5,
    title_figure=Task_Project, title_figure_fontsize=16,
    title_axes='Линейная регрессионная модель', title_axes_fontsize=14,
    x_label=Variable_Name_X,
    y_label=Variable_Name_Y,
    label_legend_regr_model=f'линейная регрессия Y = {b0:.3f} + {b1:.4f}*X',
    s=50,
    result_output=True,
    file_name='graph/regression_plot_lin.png')

Выводы и рекомендации

Исследована зависимость показаний ультразвукового прибора «ПУЛЬСАР-2.1» (X) и результатов замера прочности бетона (методом отрыва со скалыванием) склерометром ИПС-МГ4.03 (Y).

Между переменными имеется весьма сильная линейная корреляционная связь. Получена регрессионная модель:

Y = b0 + b1∙X = -21.3741 + 0.0129∙X

Модель хорошо аппроксимирует фактические данные, является адекватной, значимой и может использоваться для предсказания прочности бетона.

Также построен график прогноза с доверительными интервалами.

ИТОГИ

Итак, мы рассмотрели все этапы регрессионного анализа в случае простой линейной регрессии (simple linear regression) с использованием библиотеки statsmodels на конкретном практическом примере; подробно остановились на статистическом анализа модели с проверкой гипотез; также предложен ряд пользовательских функций, облегчающих работу исследователя и уменьшающих размер программного кода.

Конечно, мы разобрали далеко не все вопросы анализа регрессионных моделей и возможности библиотеки statsmodels применительно к simple linear regression, в частности статистики влияния (Influence Statistics), инструмент Leverage, анализ стьюдентизированных остатков и пр. — это темы для отдельных обзоров.

Исходный код находится в моем репозитории на GitHub.

Надеюсь, данный обзор поможет специалистам DataScience в работе.

Введение

В этой статье будет рассмотрен статистический метод среднеквадратичной ошибки, и я опишу связь этого метода с линией регрессии.

Пример состоит из точек на декартовой оси. Мы определим математическую функцию, которая даст нам прямую линию, которая лучше всего проходит между всеми точками на декартовой оси.

И таким образом мы узнаем связь между этими двумя методами и то, как выглядит результат их соединения вместе.

Общее объяснение

Это определение из Википедии:

В статистике среднеквадратичная ошибка (MSE) оценщика (процедуры оценки ненаблюдаемой величины) измеряет среднее квадратов ошибок, то есть среднеквадратичную разницу между оцененными значениями и тем, что оценивается. MSE — это функция риска, соответствующая ожидаемому значению квадрата потери ошибок. Тот факт, что MSE почти всегда строго положительна (а не равна нулю), объясняется случайностью или тем, что оценщик не учитывает информацию, которая могла бы дать более точную оценку.

Структура статьи

  • Получите представление об идее, графической визуализации, уравнении среднеквадратичной ошибки.
  • Математическая часть, которая содержит алгебраические манипуляции и производную функций двух переменных для нахождения минимума. Этот раздел предназначен для тех, кто хочет понять, как мы позже получаем математические формулы, вы можете пропустить его, если вас это не интересует.
  • Объяснение полученных математических формул и роли каждой переменной в формуле.
  • Примеры

Почувствуйте идею

Допустим, у нас есть семь точек, и наша цель — найти линию, которая минимизирует квадраты расстояний до этих разных точек.

Попробуем это понять.

Я возьму пример и проведу линию между точками. Конечно, мой рисунок не самый лучший, но он просто для демонстрации.

Вы можете спросить себя, что это за график?

  • фиолетовые точки — это точки на графике. Каждая точка имеет координату x и координату y.
  • Синяя линия — это линия нашего прогноза. Это линия, которая проходит через все точки и наилучшим образом подходит к ним. Эта строка содержит прогнозируемые точки.
  • красная линия между каждой фиолетовой точкой и линией прогноза — это ошибки. Каждая ошибка — это расстояние от точки до предполагаемой точки.

Вы должны помнить это уравнение еще в школьные годы, y = Mx + B, где M — это наклон линии, а B — y-точка пересечения линии.

Мы хотим найти M (наклон) и B (y-точку пересечения), которые минимизируют квадратичную ошибку!

Давайте определим математическое уравнение, которое даст нам среднеквадратичную ошибку для всех наших точек.

Давайте проанализируем, что на самом деле означает это уравнение.

  • В математике символ, который выглядит как странный E, называется суммированием (греческая сигма). Это сумма последовательности чисел от i = 1 до n. Представим это как массив точек, в котором мы перебираем все точки, от первой (i = 1) до последней (i = n).
  • Для каждой точки мы берем координату y точки и координату y’. Координата Y — это наша фиолетовая точка. Точка y находится на созданной нами линии. Мы вычитаем значение координаты Y из значения координаты Y и вычисляем квадрат результата.
  • Третья часть — взять сумму всех значений (y-y ’) ² и разделить ее на n, что даст среднее значение.

Наша цель — минимизировать это среднее, чтобы получить лучшую линию, проходящую через все точки.

От концепции к математическим уравнениям

Эта часть для людей, которые хотят понять, как мы пришли к математическим уравнениям. Если хотите, можете перейти к следующей части.

Как вы знаете, уравнение прямой имеет вид y = mx + b, где m — наклон, а b — точка пересечения с y.

Давайте возьмем каждую точку на графике, и мы сделаем наш расчет (y-y ’) ².
Но что такое y’ и как мы его вычисляем? У нас нет его в составе данных.

Но мы знаем, что для вычисления y ’нам нужно использовать наше линейное уравнение y = mx + b и поместить x в уравнение.

Отсюда получаем следующее уравнение:

Давайте перепишем это выражение, чтобы упростить его.

Давайте начнем с раскрытия всех скобок в уравнении. Я раскрасил разницу между уравнениями, чтобы облегчить понимание.

А теперь применим еще одну манипуляцию. Мы возьмем каждую часть и соберем ее вместе. Мы возьмем все y, (-2ymx) и т. Д. И поместим их все рядом.

На этом этапе мы начинаем путаться, поэтому давайте возьмем среднее значение всех квадратов значений y, xy, x, x².

Давайте определим для каждого из них новый символ, который будет представлять среднее всех квадратов значений.

Давайте посмотрим на пример. Давайте возьмем все значения y, разделим их на n, поскольку это среднее значение, и назовем его y (HeadLine).

Если мы умножим обе части уравнения на n, получим:

Это приведет нас к следующему уравнению:

Если мы посмотрим на то, что у нас получилось, мы увидим, что у нас есть трехмерная поверхность. Похоже на стакан, который резко поднимается вверх.

Мы хотим найти M и B, которые минимизируют функцию. Мы сделаем частную производную по M и частную производную по B.

Поскольку мы ищем точку минимума, мы возьмем частные производные и сравним с 0.

Давайте возьмем два полученных уравнения, выделим переменную b из обоих, а затем вычтем верхнее уравнение из нижнего.

Вычтем первое уравнение из второго.

Избавимся от знаменателей в уравнении.

Итак, это уравнение для поиска M, давайте возьмем его и запишем уравнение B.

Уравнения для наклона и пересечения по оси Y

Приведем математические уравнения, которые помогут нам найти требуемый наклон и точку пересечения оси y.

Итак, вы, вероятно, думаете про себя, что это, черт возьми, за эти странные уравнения?

На самом деле они просты для понимания, поэтому давайте поговорим о них немного.

Теперь, когда мы понимаем наши уравнения, пора собрать все воедино и показать несколько примеров.

Примеры

Большое спасибо Khan Academy за примеры.

Пример # 1

Возьмем 3 точки (1,2), (2,1), (4,3).

Найдем M и B для уравнения y = mx + b.

После того, как мы вычислили соответствующие части для нашего уравнения M и уравнения B, давайте поместим эти значения в уравнения и получим наклон и точку пересечения по оси y.

Давайте возьмем эти результаты и поместим их в линейное уравнение y = mx + b.

Теперь давайте нарисуем линию и посмотрим, как линия проходит через линии таким образом, чтобы минимизировать квадраты расстояний.

Пример # 2

Возьмем 4 балла: (-2, -3), (-1, -1), (1,2), (4,3).

Найдем M и B для уравнения y = mx + b.

Как и раньше, давайте поместим эти значения в наши уравнения, чтобы найти M и B.

Давайте возьмем эти результаты и поместим их в линейное уравнение y = mx + b.

Теперь давайте нарисуем линию и посмотрим, как линия проходит через линии таким образом, чтобы минимизировать квадраты расстояний.

В заключение

Как видите, идея проста. Нам просто нужно понять основные части и то, как мы с ними работаем.

Вы можете работать с формулами, чтобы найти линию на другом графике, а также выполнить простой расчет и получить результаты для наклона и пересечения по оси y.

Все просто, а? 😏

Приветствуются любые комментарии и отзывы — при необходимости исправлю статью.

Не стесняйтесь обращаться ко мне напрямую в LinkedIn — Щелкните здесь.

Понравилась статья? Поделить с друзьями:
  • Сдэк ошибки на сайте
  • Свыше четыре тысячи метров где ошибка
  • Свойство информации быть правильно воспринятой вероятность отсутствия ошибок
  • Сделав ошибку не бойся ее отпустить счастлив не
  • Сделали ряд решений лексическая ошибка компании